Re: [R] how to automatically create objects with names from a string list?

2008-06-04 Thread Simon Blomberg
Or with mapply

name - c(foo, bar, baz)
val - 1:3
mapply(assign, name, val, pos=1)

Cheers,

Simon.


On Tue, 2008-06-03 at 22:24 -0700, Moshe Olshansky wrote:
 You can use either assign or eval.
 
 Something like
 
  name - c(foo, bar, baz)
  for (i in 1:length(name)) assign(name[i],i)
 
 
 
 --- On Wed, 4/6/08, Mark Farnell [EMAIL PROTECTED] wrote:
 
  From: Mark Farnell [EMAIL PROTECTED]
  Subject: [R] how to automatically create objects with names from a string 
  list?
  To: R-help@r-project.org
  Received: Wednesday, 4 June, 2008, 3:15 PM
  Suppose I have a string of objects names:
  
  name - c(foo, bar,
  baz)
  
  and I would like to use a for loop to automatically create
  three
  objects called foo, bar and
  baz accordingly.  Then how can this
  be done (so that in the workspace, foo = 1, bar = 2
  and baz=3)
  
  for (i in name) {
  .
  }
  
  Thanks!
  
  Mark
  
  __
  R-help@r-project.org mailing list
  https://stat.ethz.ch/mailman/listinfo/r-help
  PLEASE do read the posting guide
  http://www.R-project.org/posting-guide.html
  and provide commented, minimal, self-contained,
  reproducible code.
 
 __
 R-help@r-project.org mailing list
 https://stat.ethz.ch/mailman/listinfo/r-help
 PLEASE do read the posting guide http://www.R-project.org/posting-guide.html
 and provide commented, minimal, self-contained, reproducible code.
-- 
Simon Blomberg, BSc (Hons), PhD, MAppStat. 
Lecturer and Consultant Statistician 
Faculty of Biological and Chemical Sciences 
The University of Queensland 
St. Lucia Queensland 4072 
Australia
Room 320 Goddard Building (8)
T: +61 7 3365 2506
http://www.uq.edu.au/~uqsblomb
email: S.Blomberg1_at_uq.edu.au

Policies:
1.  I will NOT analyse your data for you.
2.  Your deadline is your problem.

The combination of some data and an aching desire for 
an answer does not ensure that a reasonable answer can 
be extracted from a given body of data. - John Tukey.

__
R-help@r-project.org mailing list
https://stat.ethz.ch/mailman/listinfo/r-help
PLEASE do read the posting guide http://www.R-project.org/posting-guide.html
and provide commented, minimal, self-contained, reproducible code.


Re: [R] merging two data sets with no column header

2008-06-04 Thread David Winsemius

Thanks for your reply. 

In your last step If  I create a duplicate ( two similar records )
# create a duplicate 
vv[8,1] - 7
vv[8,2]-'g'

and then I merge vv with vv2 ,both duplicates are merged. Is there a way to
tell R to merge only the unique records.

--

I don't think merge will do what you are asking. Suspect you would need to
do it either before the merge or after. I tried using the Extract operator
on a merged dataset, but only succeeded in omitting the duplicated items, 
not both instances.

-- 
David Winsemius



-- 
View this message in context: 
http://www.nabble.com/merging-two-data-sets-with-no-column-header-tp17613296p17639592.html
Sent from the R help mailing list archive at Nabble.com.

__
R-help@r-project.org mailing list
https://stat.ethz.ch/mailman/listinfo/r-help
PLEASE do read the posting guide http://www.R-project.org/posting-guide.html
and provide commented, minimal, self-contained, reproducible code.


[R] ignoring environment value of R_HOME error when installing packages

2008-06-04 Thread tub78

I am troubled by what appears to be a glitch in the current
distribution, or in
its installation on our system.  I've traced it, and found a work-
around.  Is
this normal?  Is there a cleaner solution?


The problem:

During a package installation, the warning message WARNING: ignoring
environment value of R_HOME from line 31 of R_HOME/bin/R is
accidentally
spliced into the CLINK_CPPFLAGS variable at line 606 of R_HOME/bin/
INSTALL.
This prevents the compilation C files.  Please note that I have not
set the R_HOME
environment variable; it is undefined in my shell.

- Here are lines 29-32 from R_HOME/bin/R:

if test -n ${R_HOME}  \
   test ${R_HOME} != ${R_HOME_DIR}; then
  echo WARNING: ignoring environment value of R_HOME
fi

- Here is line 606 of R_HOME/bin/INSTALL:

CLINK_CPPFLAGS=`echo
tools:::.find_cinclude_paths(file='DESCRIPTION') | \
  ${R_EXE} --vanilla --slave`



The work-around:

First some background.  The command sequence R CMD INSTALL sets in
motion a sequence of scripts that collectively manage the installation
process.

RR_HOME/bin/R  calls Rcmd script on line
148
CMD  R_HOME/lib/R/bin/Rcmd calls INSTALL script on
line 45
INSTALL  R_HOME/lib/R/bin/INSTALL  encounters the error on
line 606

Now, if you are the owner of you installation, then you can probably
just edit
the INSTALL file directly.

Otherwise, if you specify the full path of an executable in place of
the word
INSTALL on the command line, the Rcmd script will detect this and
call your
script instead of the normal INSTALL script (see Rcmd line 37).

e.g.  R CMD path.to.alternate.install.script ...

One can just copy the INSTALL script, changing line 606 to
CLINK_CPPFLAGS = 
and hope for the best.  If the package you are compiling specifies its
own C
include files, then you will have to modify the variable accordingly,
or else
use the ~/.R/Makevars mechanism.

But, the question remains, is there an nicer solution?

Thanks,
- Stu

__
R-help@r-project.org mailing list
https://stat.ethz.ch/mailman/listinfo/r-help
PLEASE do read the posting guide http://www.R-project.org/posting-guide.html
and provide commented, minimal, self-contained, reproducible code.


Re: [R] R-2.7.0 make check failure

2008-06-04 Thread Prof Brian Ripley
This indicates a serious problem with your build (a segfault). 
Unfortunately as R is very well tested on Linux and we have never seen 
this one reported, we have no clue as to why.  You've told us very little 
(what architecture, what compilers?) so although it seems to be something 
specific to your OS/machine, you will need to find out what via the 
debugger.


The most likely guess as to the cause is a compiler optimization error, so 
I would try building without optimization.


On Tue, 3 Jun 2008, Gregory Ruchti wrote:


Hello,

I am fairly new to using R and am trying to install it on my Linux machine, 
running Scientific Linux.  I get through running 'configure' and 'make' OK, 
but when I run 'make check', I get the following error:


make check
make[1]: Entering directory `/home/gruchti/Programs/R-2.7.0/tests'
make[2]: Entering directory `/home/gruchti/Programs/R-2.7.0/tests'
make[3]: Entering directory `/home/gruchti/Programs/R-2.7.0/tests/Examples'
make[4]: Entering directory `/home/gruchti/Programs/R-2.7.0/tests/Examples'
make[4]: `Makedeps' is up to date.
make[4]: Leaving directory `/home/gruchti/Programs/R-2.7.0/tests/Examples'
make[4]: Entering directory `/home/gruchti/Programs/R-2.7.0/tests/Examples'
running code in 'base-Ex.R' .../bin/sh: line 1:  4233 Segmentation fault 
../../bin/R --vanilla base-Ex.R base-Ex.Rout 21

make[4]: *** [base-Ex.Rout] Error 1
make[4]: Leaving directory `/home/gruchti/Programs/R-2.7.0/tests/Examples'
make[3]: *** [test-Examples-Base] Error 2
make[3]: Leaving directory `/home/gruchti/Programs/R-2.7.0/tests/Examples'
make[2]: *** [test-Examples] Error 2
make[2]: Leaving directory `/home/gruchti/Programs/R-2.7.0/tests'
make[1]: *** [test-all-basics] Error 1
make[1]: Leaving directory `/home/gruchti/Programs/R-2.7.0/tests'
make: *** [check] Error 2

I took a look at the 'base-Ex.Rout.fail' file to see where the problem 
occured and get the following at the end of the file:



## R code version of choose()  [simplistic; warning for k  0]:
mychoose - function(r,k)

+ ifelse(k = 0, (k==0),
+sapply(k, function(k) prod(r:(r-k+1))) / factorial(k))

k - -1:6
cbind(k=k, choose(1/2, k), mychoose(1/2, k))


*** caught segfault ***
address 0x200, cause 'memory not mapped'

Traceback:
1: doWithOneRestart(return(expr), restart)
2: withOneRestart(expr, restarts[[1]])
3: withRestarts({.Internal(.signalCondition(simpleWarning(msg, call), 
msg, call)).Internal(.dfltWarn(msg, call))}, muffleWarning = 
function() NULL)

4: .signalSimpleWarning(NaNs produced, quote(gamma(x + 1)))
5: factorial(k)
6: ifelse(k = 0, (k == 0), sapply(k, function(k) prod(r:(r - k + 
1)))/factorial(k))

7: mychoose(1/2, k)
8: cbind(k = k, choose(1/2, k), mychoose(1/2, k))
aborting ...


I really am not sure how to interpret this, or how to fix it.  Any help would 
be greatly appreciated!


Regards,
Greg

--
Gregory Ruchti
Bloomberg Center for Physics and Astronomy
Johns Hopkins University

__
R-help@r-project.org mailing list
https://stat.ethz.ch/mailman/listinfo/r-help
PLEASE do read the posting guide http://www.R-project.org/posting-guide.html
and provide commented, minimal, self-contained, reproducible code.


--
Brian D. Ripley,  [EMAIL PROTECTED]
Professor of Applied Statistics,  http://www.stats.ox.ac.uk/~ripley/
University of Oxford, Tel:  +44 1865 272861 (self)
1 South Parks Road, +44 1865 272866 (PA)
Oxford OX1 3TG, UKFax:  +44 1865 272595

__
R-help@r-project.org mailing list
https://stat.ethz.ch/mailman/listinfo/r-help
PLEASE do read the posting guide http://www.R-project.org/posting-guide.html
and provide commented, minimal, self-contained, reproducible code.


Re: [R] unable to get plot in R

2008-06-04 Thread Prof Brian Ripley

On Tue, 3 Jun 2008, Duncan Murdoch wrote:


On 03/06/2008 6:53 PM, Chin-Cheng Su wrote:

Hi everyone,

I use R to plot my data set (x,y) in Windows XP, but keep getting error 
message like follows,



...
plot(x, y)

Error in windows() : unable to start device devWindows

How can I solve this problem? Thanks for any suggestion or help.


Somehow you appear to have set options(device) to devWindows.  If you set 
it to windows using options(device=windows), things should be fine.


I don't think so.  windows() is a family of devices, and that is what they 
report if GADeviceDriver fails.  That would seem to mean that either 
allocation fails (unlikely) or GA_Open fails (which AFAICS only fails if 
newwindow does).


We don't have the 'at a minimum' details requested in the posting guide, 
and this might be something to do with the locale.  I'd suggest starting R 
in an English locale if that is not already the case (add LC_ALL=en to the 
target of the shortcut used to start R -- see the rw-FAQ).


--
Brian D. Ripley,  [EMAIL PROTECTED]
Professor of Applied Statistics,  http://www.stats.ox.ac.uk/~ripley/
University of Oxford, Tel:  +44 1865 272861 (self)
1 South Parks Road, +44 1865 272866 (PA)
Oxford OX1 3TG, UKFax:  +44 1865 272595

__
R-help@r-project.org mailing list
https://stat.ethz.ch/mailman/listinfo/r-help
PLEASE do read the posting guide http://www.R-project.org/posting-guide.html
and provide commented, minimal, self-contained, reproducible code.


[R] problems with variogram

2008-06-04 Thread Laura Saltyte
Hello,

I have data at 10 locations, in each location there are time series
(T=56). Question is: when I’m fitting variogram what happens with those
measures in each location? Are they taken as repeated measures? It's very
important for my to know this

Thanks a lot

__
R-help@r-project.org mailing list
https://stat.ethz.ch/mailman/listinfo/r-help
PLEASE do read the posting guide http://www.R-project.org/posting-guide.html
and provide commented, minimal, self-contained, reproducible code.


Re: [R] Model simplification using anova()

2008-06-04 Thread Dieter Menne
ChCh jmo101 at student.canterbury.ac.nz writes:

 
 
 Hello all,
 
 I've become confused by the output produced by a call to
 anova(model1,model2). First a brief background.  My model used to predict
 final tree height is summarised here:
 
 Df Sum Sq Mean Sq F valuePr(F)
 Treatment  2   748.35   374.17 21.3096   7.123e-06 ***
 HeightInitial1   0.31  0.31 0.01780.89519
 DiameterInitial 1   0.52  0.52 0.02980.86460
 Frost 1   38.2938.292.18070.15392
 HeightInitial:Frost1   85.83 85.83   4.88820.03774 *  
 DiameterInitial:Frost 1   97.9097.905.57540.02749 *  
 Residuals22 386.30   17.56  
 ---
 
 Based on this, I should not remove either of the interaction terms, so I
 turned my attention to the main factors. Based on p-values, I removed
 HeightInitial and used a call to anova(model1,model2) to see if this
 resulted in a weaker model.  Here is the output:
...

Though should never remove main factors in the presence of interactions. Check
Bill Venables penitential sermon which quite on top of google when you enter
exegeses.

http://www.stats.ox.ac.uk/pub/MASS3/Exegeses.pdf

The usual way to simplify models in R is based on the AIC, not on p-values. See
stepAIC and the chapter in MASS.

Dieter

__
R-help@r-project.org mailing list
https://stat.ethz.ch/mailman/listinfo/r-help
PLEASE do read the posting guide http://www.R-project.org/posting-guide.html
and provide commented, minimal, self-contained, reproducible code.


[R] random time serie

2008-06-04 Thread jessica . gervais

Dear all,

I would like to create an random time serie of POSIXct elements inside a
certain range of dates and time.
For example a sequences of date/time of 400 elements, randomly choosen,
with all dates between 2002-12-01 and 2002-12-31 and all time between
07:00:00 and 23:00:00.

Would anyone know if there is a specific R - function for that ?

Many thanks in advance,

Jessica

_

Jessica Gervais
Tel: +352- 425991-628
Mail: [EMAIL PROTECTED]

Resource Centre for Environmental Technologies,
Public Research Centre Henri Tudor,
Technoport Schlassgoart,
66 rue de Luxembourg,
P.O. BOX 144,
L-4002 Esch-sur-Alzette, Luxembourg

__
R-help@r-project.org mailing list
https://stat.ethz.ch/mailman/listinfo/r-help
PLEASE do read the posting guide http://www.R-project.org/posting-guide.html
and provide commented, minimal, self-contained, reproducible code.


Re: [R] matlab eigs function in R

2008-06-04 Thread Frede Aakmann Tøgersen
Thomas

As far I can see eig (matlab) and eigen (R) both uses LAPACK as the engine, so 
no differences apart from rounding errors should be expected. 

You are not being particular friendly, since you have not told the whole story 
about how you use eigs in matlab (you are giving some options to eigs). The man 
page for eigs tells that eigs find largest eigenvalues and eigenvectors of a 
sparse matrix. However to see how this is done one needs to consult the 
references (see below) in order to find out. It is some iterative method 
depending on numerous parameters.

If I use eigs with default values for it's options on 


M = [1.2607   -3.55752.2968 0 0 0
-3.5575   10.1429   -6.5855 0 0 0
 2.2968   -6.58554.2887 0 0 0
  0 0 02.6359   -4.84892.2130
  0 0 0   -4.84898.9217   -4.0728
  0 0 02.2130   -4.07281.8598]


to find the four eigenvalues closest to 0 I get:

[Y,eigenvals] = eigs(M,4,0)
Iteration 1: a few Ritz values of the 6-by-6 matrix:
 0
 0
 0
 0


Y =

   0.77166670795421  -0.7309382085  -0.00  -0.57709401467465
  -0.15442436854521   0.00132216402668  -0.01  -0.57751272471495
  -0.61675233485287  -0.00113088742149  -0.00  -0.57744396127772
   0.00795845605741  -0.68639286168232   0.57735026918974   0.1228756964
  -0.01410736097199  -0.03974888299631   0.57735026918963  -0.00011191813149
   0.00614890491458   0.72614174467891   0.57735026918951   0.9963056184


eigenvals =

   0.02530485525437  0  0  0
  0   0.00109157646361  0  0
  0  0  -0.00  0
  0  0  0  -0.3332911697



Now setting e.g.

options.issym = 1
options.p = 6 (number of basis vectors)

we now get

[Y,eigenvals] = eigs(M,4,0,options)
Iteration 1: a few Ritz values of the 6-by-6 matrix:
 0
 0
 0
 0


Y =

  -0.76500157409670  -0.00223987987076  -0.00  -0.57702216553796
   0.13515933807820   0.00165998521603  -0.00  -0.57744188181803
   0.62913726290127   0.00151722712519  -0.00  -0.57758630415522
  -0.00973169974519  -0.68255505830793   0.57735026918973  -0.00048056999418
   0.02143072866845  -0.04677389396937   0.57735026918963   0.00017535669432
  -0.01169902892326   0.72932895227757   0.57735026918951   0.00030521329984


eigenvals =

   0.02542832424752  0  0  0
  0   0.00110871605179  0  0
  0  0  -0.00  0
  0  0  0  -0.3331498129

Different answers!!!

What causes the difference the references should explain to us.


If you need functions for sparse matrices in R consider the sparseM and Matrix 
packages. The latter has a eigen() but I don't know which methods it uses to 
calculate the eigenvalues and -vectors.


Best regards

Frede Aakmann Tøgersen
Scientist


UNIVERSITY OF AARHUS
Faculty of Agricultural Sciences
Dept. of Genetics and Biotechnology
Blichers Allé 20, P.O. BOX 50
DK-8830 Tjele

Phone:   +45 8999 1900
Direct:  +45 8999 1878

E-mail:  [EMAIL PROTECTED]
Web:   http://www.agrsci.org

This email may contain information that is confidential.
Any use or publication of this email without written permission from Faculty of 
Agricultural Sciences is not allowed.
If you are not the intended recipient, please notify Faculty of Agricultural 
Sciences immediately and delete this email.

 

References for eigs



[1]  Lehoucq, R.B. and D.C. Sorensen, Deflation Techniques for an Implicitly 
Re-Started Arnoldi Iteration, SIAM J. Matrix Analysis and Applications, Vol. 
17, 1996, pp. 789-821. 

[2]  Lehoucq, R.B., D.C. Sorensen, and C. Yang, ARPACK Users' Guide: Solution 
of Large-Scale Eigenvalue Problems with Implicitly Restarted Arnoldi Methods, 
SIAM Publications, Philadelphia, 1998. 

[3]  Sorensen, D.C., Implicit Application of Polynomial Filters in a k-Step 
Arnoldi Method, SIAM J. Matrix Analysis and Applications, Vol. 13, 1992, pp. 
357-385. 





 

 -Oprindelig meddelelse-
 Fra: [EMAIL PROTECTED] 
 [mailto:[EMAIL PROTECTED] På vegne af kayteck_master
 Sendt: 3. juni 2008 8:22
 Til: r-help@r-project.org
 Emne: Re: [R] matlab eigs function in R
 
 
 Dear Thomas
 
 Yes, you're right. But I'm looking for this not only for 
 computational cost reasons. 
 eigen function from R has identical behavior to matlab 
 function eig, but not to eigs.
 Even in matlab you can check that those function are not 
 giving identical results.
 Here you have a short example :
 Suppose we have given a 

[R] missing data imputation - simulation

2008-06-04 Thread Birgitle

My dataset contains missing data and I would like to do something like an EM
algorithm or a Markov Chain Monte Carlo approach to get rid of the missing
data.

Is there a function for imputation or simulation of missing data apart from
those in the randomForest library?

Thanks in advance

Birgit

-
The art of living is more like wrestling than dancing.
(Marcus Aurelius)
-- 
View this message in context: 
http://www.nabble.com/missing-data-imputation---simulation-tp17642736p17642736.html
Sent from the R help mailing list archive at Nabble.com.

__
R-help@r-project.org mailing list
https://stat.ethz.ch/mailman/listinfo/r-help
PLEASE do read the posting guide http://www.R-project.org/posting-guide.html
and provide commented, minimal, self-contained, reproducible code.


Re: [R] splitting data frame based on a criteria

2008-06-04 Thread Petr PIKAL
Hi

the other possible option is to use split with suitable graphing technique 
or with lapply and appropriate function
e.g.

boxplot(split(X, Y0))
histogram( ~ X|Y)

gives Marvin 

[EMAIL PROTECTED] napsal dne 03.06.2008 21:11:29:

 ?by may be helpful here
 eg if dat is your data.frame and yf is a factor (created using ifelse)
 use by(dat,yf,mean) to compute the means for each level of yf
 hth, Ingmar
 
 On Jun 3, 2008, at 8:37 PM, Marvin Lists wrote:
 
  Hi,
  I have a data frame that I want to split into two based on the 
  values of a
  variable in it.
 
  The variable Y has numeric values ranging between 0 through 70. I 
  want to
  plot the frequencies of another variable X in two different cases:
  - When Y = 0 and
  - When Y  0
 
  How does one go about doing this?
 
  In general, I want to do several analyses with this data frame that 
  are a
  variation of the above situation, i.e. they require splitting the 
  data into
  different age, gender etc. and then calculating separate means, 
  correlations
  and so on for the different groups into which the data frame would 
  split.
 
  I am struggling with the correct syntax for achieving this.
 
  Reading through the documentation suggests that tapply and split 
  may be the
  functions to use for my purposes but the examples in the 
  documentations
  didn't help me understand how I could achieve this.
 
  I would appreciate any suggestions and help.
 
  Thanks,
  Marvin
 
 [[alternative HTML version deleted]]
 
  __
  R-help@r-project.org mailing list
  https://stat.ethz.ch/mailman/listinfo/r-help
  PLEASE do read the posting guide http://www.R-project.org/posting- 
  guide.html
  and provide commented, minimal, self-contained, reproducible code.
 
 __
 R-help@r-project.org mailing list
 https://stat.ethz.ch/mailman/listinfo/r-help
 PLEASE do read the posting guide 
http://www.R-project.org/posting-guide.html
 and provide commented, minimal, self-contained, reproducible code.

__
R-help@r-project.org mailing list
https://stat.ethz.ch/mailman/listinfo/r-help
PLEASE do read the posting guide http://www.R-project.org/posting-guide.html
and provide commented, minimal, self-contained, reproducible code.


[R] Odp: column and row

2008-06-04 Thread Petr PIKAL
Hi

use 
str(object)
to see what is a structure of object
then use standard selecting by [..., ...]

In your case you shall probably use extracting function

coef(fit1)[1,] 
for the first row and similarly for the second.

Regards
Petr
[EMAIL PROTECTED]
724008364, 581252140, 581252257


[EMAIL PROTECTED] napsal dne 04.06.2008 03:55:47:

 -fit1$coefficients
 
  y.control y.lowy.high
 (Intercept)19.628713 21.883999 20.023814
 log(1 + (age - 45)/10) -7.383725 -6.017342 -5.084431
 
 here is my outcome,I need one vector say b1=first row without the 
intercept
 ,like:(19.628713, 21.883999, 20.023814)
 and  another b2=second row without the log(1 + (age - 45)/10),only the
 data,how to do this in R?
 
[[alternative HTML version deleted]]
 
 __
 R-help@r-project.org mailing list
 https://stat.ethz.ch/mailman/listinfo/r-help
 PLEASE do read the posting guide 
http://www.R-project.org/posting-guide.html
 and provide commented, minimal, self-contained, reproducible code.

__
R-help@r-project.org mailing list
https://stat.ethz.ch/mailman/listinfo/r-help
PLEASE do read the posting guide http://www.R-project.org/posting-guide.html
and provide commented, minimal, self-contained, reproducible code.


[R] data frame titles

2008-06-04 Thread Chris Arthur
I'm new to R so please forgive the newbie question; but i can't seem to 
find a definitive answer to this.


I am wanting to do a PLS regression on some data. Which takes a formula 
of the type

responseACC ~ dataACC

where dataACC is multivariate in nature and responseACC is a single value.

I have imported my data from a csv file into a dataframe called for 
arguments sake df.

This file has no headers in it so I get column names X1.X83

I then create a new data frame which says
data.frame(dataACC = df[1:82], responseACC = df[83])

When I look at the names of this though I get...

dataACC.X1 . dataACC.X82 and then X83

When I pass this to the PLS algorithm I get the error

variable responseACC not found

So a few questions, Am I doing this in the right way in general? why is 
the responseACC not being associated as the name of df[83]?


Thirdly, If I want to generate my own array of values in order to do get 
a predicted response based on my initial PLS regression, will it matter 
that the array data will not have header values (eg dataACC.X1 etc)? if 
so what the best way of appending the header data?


Thanks for any help you can lend

Chris

__
R-help@r-project.org mailing list
https://stat.ethz.ch/mailman/listinfo/r-help
PLEASE do read the posting guide http://www.R-project.org/posting-guide.html
and provide commented, minimal, self-contained, reproducible code.


Re: [R] missing data imputation - simulation

2008-06-04 Thread Chuck Cleland

On 6/4/2008 5:32 AM, Birgitle wrote:

My dataset contains missing data and I would like to do something like an EM
algorithm or a Markov Chain Monte Carlo approach to get rid of the missing
data.

Is there a function for imputation or simulation of missing data apart from
those in the randomForest library?

Thanks in advance

Birgit


RSiteSearch(imputation, restrict=functions)

RSiteSearch({multiple imputation}, restrict=functions)

  return many relevant hits.


-
The art of living is more like wrestling than dancing.
(Marcus Aurelius)


--
Chuck Cleland, Ph.D.
NDRI, Inc. (www.ndri.org)
71 West 23rd Street, 8th floor
New York, NY 10010
tel: (212) 845-4495 (Tu, Th)
tel: (732) 512-0171 (M, W, F)
fax: (917) 438-0894

__
R-help@r-project.org mailing list
https://stat.ethz.ch/mailman/listinfo/r-help
PLEASE do read the posting guide http://www.R-project.org/posting-guide.html
and provide commented, minimal, self-contained, reproducible code.


Re: [R] missing data imputation - simulation

2008-06-04 Thread Ulrike Grömping

Birgit,

not knowing your data, I would recommend R-package mice or function
aregImpute from R-package Hmisc as good multi-purpose tools.

Regards, Ulrike

-- 
View this message in context: 
http://www.nabble.com/missing-data-imputation---simulation-tp17642736p17643601.html
Sent from the R help mailing list archive at Nabble.com.

__
R-help@r-project.org mailing list
https://stat.ethz.ch/mailman/listinfo/r-help
PLEASE do read the posting guide http://www.R-project.org/posting-guide.html
and provide commented, minimal, self-contained, reproducible code.


[R] A little test -- please forgive!

2008-06-04 Thread Ted Harding
Sorry for the noise -- but I want to see what happens
to this message when it gets to tyhe archives.
Ted.


E-Mail: (Ted Harding) [EMAIL PROTECTED]
Fax-to-email: +44 (0)870 094 0861
Date: 04-Jun-08   Time: 11:02:57
-- XFMail --

__
R-help@r-project.org mailing list
https://stat.ethz.ch/mailman/listinfo/r-help
PLEASE do read the posting guide http://www.R-project.org/posting-guide.html
and provide commented, minimal, self-contained, reproducible code.


Re: [R] missing data imputation - simulation

2008-06-04 Thread Birgitle

Many thenks to both of you:
Will have a look.

Birgit


Chuck Cleland wrote:
 
 On 6/4/2008 5:32 AM, Birgitle wrote:
 My dataset contains missing data and I would like to do something like an
 EM
 algorithm or a Markov Chain Monte Carlo approach to get rid of the
 missing
 data.
 
 Is there a function for imputation or simulation of missing data apart
 from
 those in the randomForest library?
 
 Thanks in advance
 
 Birgit
 
 RSiteSearch(imputation, restrict=functions)
 
 RSiteSearch({multiple imputation}, restrict=functions)
 
return many relevant hits.
 
 -
 The art of living is more like wrestling than dancing.
 (Marcus Aurelius)
 
 -- 
 Chuck Cleland, Ph.D.
 NDRI, Inc. (www.ndri.org)
 71 West 23rd Street, 8th floor
 New York, NY 10010
 tel: (212) 845-4495 (Tu, Th)
 tel: (732) 512-0171 (M, W, F)
 fax: (917) 438-0894
 
 __
 R-help@r-project.org mailing list
 https://stat.ethz.ch/mailman/listinfo/r-help
 PLEASE do read the posting guide
 http://www.R-project.org/posting-guide.html
 and provide commented, minimal, self-contained, reproducible code.
 
 


-
The art of living is more like wrestling than dancing.
(Marcus Aurelius)
-- 
View this message in context: 
http://www.nabble.com/missing-data-imputation---simulation-tp17642736p17644180.html
Sent from the R help mailing list archive at Nabble.com.

__
R-help@r-project.org mailing list
https://stat.ethz.ch/mailman/listinfo/r-help
PLEASE do read the posting guide http://www.R-project.org/posting-guide.html
and provide commented, minimal, self-contained, reproducible code.


[R] Comparing two regression lines

2008-06-04 Thread Christoph Scherber

Dear R users,

Suppose I have two different response variables y1, y2 that I regress separately on the same 
explanatory variable, x; sample sizes are n1=n2.


Is it legitimate to compare the regression slopes (equal variances assumed) by 
using

lm(y~x*FACTOR),

where FACTOR gets y1 if y1 is the response, and y2 if y2 is the response?

The problem I see here is that the residual degrees of freedom obviously go up (n1-1+n2-1) although 
there were only n1=n2 true replications.


On the other hand, Zar (1984) and some other statistics textbooks base their calculations on a 
modified version of the t test (mainly using the residual SS from the two regressions), but when I 
calculated these tests by hand I found that the t-values are almost identical to the one I get using 
the lm(...) approach.


Part of the R script is appended below.

How would you proceed?

Many thanks for your help!

Best wishes,
Christoph

##

lm1=lm(y1~x) # y1 and y2 are different response variables, scaled to [0;1] 
using a ranging transform
lm2=lm(y2~x) # n is 12 for each regression

model3=lm(y~x*FACTOR)
summary(model3)

sxx1=sum((y1-mean(y1))^2)
sxx2=sum((y2-mean(y2))^2)

s.pooled=(4805.2+6946.6)/20

SED=sqrt(s.pooled*(1/sxx1+1/sxx2))

t=(528.54-446.004)/SED #residual SS from lm1 and lm2 divided by SED
qt(0.025,20) #compare with t distribution at 20 d.f.


##
#(using R 2.6.2 on Windows XP)

__
R-help@r-project.org mailing list
https://stat.ethz.ch/mailman/listinfo/r-help
PLEASE do read the posting guide http://www.R-project.org/posting-guide.html
and provide commented, minimal, self-contained, reproducible code.


[R] Odp: data frame titles

2008-06-04 Thread Petr PIKAL
Hi

[EMAIL PROTECTED] napsal dne 04.06.2008 10:57:26:

 I'm new to R so please forgive the newbie question; but i can't seem to 
 find a definitive answer to this.
 
 I am wanting to do a PLS regression on some data. Which takes a formula 
 of the type
 responseACC ~ dataACC
 
 where dataACC is multivariate in nature and responseACC is a single 
value.

Really? From which package? AFAIK in package pls you have to enter formula
plsr(response~terms, data.frame)

 
 I have imported my data from a csv file into a dataframe called for 
 arguments sake df.
 This file has no headers in it so I get column names X1.X83
 
 I then create a new data frame which says
 data.frame(dataACC = df[1:82], responseACC = df[83])

You probably want
data.frame(dataACC = df[,1:82], responseACC = df[,83])

 
 When I look at the names of this though I get...
 
 dataACC.X1 . dataACC.X82 and then X83
 
 When I pass this to the PLS algorithm I get the error
 
 variable responseACC not found
 
 So a few questions, Am I doing this in the right way in general? why is 
 the responseACC not being associated as the name of df[83]?
 
 Thirdly, If I want to generate my own array of values in order to do get 

 a predicted response based on my initial PLS regression, will it matter 
 that the array data will not have header values (eg dataACC.X1 etc)? if 
 so what the best way of appending the header data?

see ?names

Regards
Petr

 
 Thanks for any help you can lend
 
 Chris
 
 __
 R-help@r-project.org mailing list
 https://stat.ethz.ch/mailman/listinfo/r-help
 PLEASE do read the posting guide 
http://www.R-project.org/posting-guide.html
 and provide commented, minimal, self-contained, reproducible code.

__
R-help@r-project.org mailing list
https://stat.ethz.ch/mailman/listinfo/r-help
PLEASE do read the posting guide http://www.R-project.org/posting-guide.html
and provide commented, minimal, self-contained, reproducible code.


Re: [R] Minimizing the negative log likelihood function...

2008-06-04 Thread Karl Ove Hufthammer
François Aucoin:

 I have tried several R's functions for optimization but the results I
 yield are not correct.
 Is there anybody who can help me?

I couldn't get it to estimate the correct values either, so I guess either
your random number generator 'rkappa' is wrong, or your 'Neg.Log.Like'
function is wrong. Or possibly both.

-- 
Karl Ove Hufthammer

__
R-help@r-project.org mailing list
https://stat.ethz.ch/mailman/listinfo/r-help
PLEASE do read the posting guide http://www.R-project.org/posting-guide.html
and provide commented, minimal, self-contained, reproducible code.


[R] Using library in an example

2008-06-04 Thread Jim Lemon

Hi all,
I've been trying to include a nice example in the Rd file for a function 
and cannot work out how to get it through R check. If I use:


if(require(maps,quietly=TRUE)) {
 ...

Warning message:
'Sys.putenv' is deprecated.
Use 'Sys.setenv' instead.
See help(Deprecated)

The warning message causes R check to get the tremors and assume there 
is something wrong with the example. This also means that I can't put a 
little message telling the user that they needed maps to run the 
example, as the warning message leads to the else being detached from 
the if (and another tremor and error message).


I can't really use the following, even though it doesn't cause the 
warning (but would be messy if the user didn't have the package).


if(library(maps)) {
 ...

If I try:

if(library(maps,logical.return=TRUE)) {
 ...

I get the same warning as with require. Does this qualify as a bug in 
require/library or should I just not bother to test the result of 
library and let the user suffer?


Jim

__
R-help@r-project.org mailing list
https://stat.ethz.ch/mailman/listinfo/r-help
PLEASE do read the posting guide http://www.R-project.org/posting-guide.html
and provide commented, minimal, self-contained, reproducible code.


[R] not meaningful for factors

2008-06-04 Thread Philip Twumasi-Ankrah
I am trying to define groupings from levels of factor variables and this the 
warning message that R give 

  not meaningful for factors.

The nature of my task is this. I have a variable stage which has the levels 
(1B, 2A, 2B) - these are the AJCC TNM stages of cancer, and another variable 
diameter with factor levels (= 4, 4 - 6.5,  6.5; limit values are 
exclusive). 

I am trying to define series of groupings based on these variables and others 
like them. 

My first attempts were;

1. node1  - data.trt[data.trt$stage==1B   data.trt$diameter=== 4] 
2.  data.trt$stage[data.trt$stage==1B] 
data.trt$diameter[data.trt$diameter=== 4]

The second attempt was purely a fishing exercise.

R gave me the waring message:

Warning message:
In Ops.factor(data.trt$stage[data.trt$stage == 1B], 
data.trt$diameter[data.trt$diameter ==  :
   not meaningful for factors


My question is how do I get round this and by implication is there an 
alternative way of defining a logical operation applicable to factors.

Thanks already for the help.

Philip


A Smile costs Nothing  

 But Rewards Everything

Happiness is not perfected until it is shared
  -Jane Porter  
  


   
[[alternative HTML version deleted]]

__
R-help@r-project.org mailing list
https://stat.ethz.ch/mailman/listinfo/r-help
PLEASE do read the posting guide http://www.R-project.org/posting-guide.html
and provide commented, minimal, self-contained, reproducible code.


Re: [R] request: An array declarion problem

2008-06-04 Thread jim holtman
When I run your code, I get a different error:

 j=10; ss=150; r=array(0 , c( j , ss )); rr=array(0 , c( j , ss ));
r1=array(0 , c( j-1 , ss )); r2=array(0 , c( j-1 , ss ));
 r3=array(0 , c( 2 , ss ))

 for(i in 1:j-1){
+r1[ i , ] - r[ j+1, ]-r[ j, ]; r2[ i , ] - rr[
j+1, ]-rr[ j, ]
+r3[ i,]=rbind(r1[ i, ], r2[ i, ])
+}
Error: subscript out of bounds

This is probably due to 'r' being dimensioned (j,ss) and you are referencing
'r[j+1,]' which is r[11,].  Also I think you want for(i in 1:(j-1)),
notice the difference:

 1:j-1
 [1] 0 1 2 3 4 5 6 7 8 9
 1:(j-1)
[1] 1 2 3 4 5 6 7 8 9




On Tue, Jun 3, 2008 at 1:58 PM, Muhammad Azam [EMAIL PROTECTED] wrote:

 Dear R users
 I tried a lot to solve the following problem but could not. I have two
 arrays having same order i.e 1 by 150.

 j=10; ss=150; r=array(0 , c( j , ss )); rr=array(0 , c( j , ss ));
 r1=array(0 , c( j-1 , ss )); r2=array(0 , c( j-1 , ss ));
 r3=array(0 , c( 2 , ss ))

 for(i in 1:j-1){
r1[ i , ] - r[ j+1, ]-r[ j, ]; r2[ i , ] - rr[
 j+1, ]-rr[ j, ]
}

 Now i want to rbind the results of r1 and r2 for each time. Now the order
  of r3[ j, ] will become 2 by 150. I used the following form in the loop

 r3[ i,]=rbind(r1[ i, ], r2[ i, ])

 But there is an error message

 Error in r3[i, ] = rbind(r1[i, ], r2[i, ]) :  number of items to replace is
 not a multiple of replacement length


 I am looking for some suggestion to solve the problem. Thanks and


 best regards

 Muhammad Azam
 Ph.D. Student
 Department of Medical Statistics,
 Informatics and Health Economics
 University of Innsbruck, Austria



[[alternative HTML version deleted]]

 __
 R-help@r-project.org mailing list
 https://stat.ethz.ch/mailman/listinfo/r-help
 PLEASE do read the posting guide
 http://www.R-project.org/posting-guide.htmlhttp://www.r-project.org/posting-guide.html
 and provide commented, minimal, self-contained, reproducible code.




-- 
Jim Holtman
Cincinnati, OH
+1 513 646 9390

What is the problem you are trying to solve?

[[alternative HTML version deleted]]

__
R-help@r-project.org mailing list
https://stat.ethz.ch/mailman/listinfo/r-help
PLEASE do read the posting guide http://www.R-project.org/posting-guide.html
and provide commented, minimal, self-contained, reproducible code.


Re: [R] not meaningful for factors

2008-06-04 Thread jim holtman
What exactly are you trying to do?  In the first case you are making a
logical comparison and that is legal for .  In the second you are trying
to do a logical operation () between two factors and that operation is
not defined.  This is what the error message is saying.

Also you first attempt is probably missing a comma:

 node1  - data.trt[data.trt$stage==1B   data.trt$diameter=== 4, ]


On Wed, Jun 4, 2008 at 8:10 AM, Philip Twumasi-Ankrah 
[EMAIL PROTECTED] wrote:

 I am trying to define groupings from levels of factor variables and this
 the warning message that R give

   not meaningful for factors.

 The nature of my task is this. I have a variable stage which has the levels
 (1B, 2A, 2B) - these are the AJCC TNM stages of cancer, and another variable
 diameter with factor levels (= 4, 4 - 6.5,  6.5; limit values are
 exclusive).

 I am trying to define series of groupings based on these variables and
 others like them.

 My first attempts were;

 1. node1  - data.trt[data.trt$stage==1B   data.trt$diameter=== 4]
 2.  data.trt$stage[data.trt$stage==1B]
 data.trt$diameter[data.trt$diameter=== 4]

 The second attempt was purely a fishing exercise.

 R gave me the waring message:

 Warning message:
 In Ops.factor(data.trt$stage[data.trt$stage == 1B],
 data.trt$diameter[data.trt$diameter ==  :
   not meaningful for factors


 My question is how do I get round this and by implication is there an
 alternative way of defining a logical operation applicable to factors.

 Thanks already for the help.

 Philip


 A Smile costs Nothing

 But Rewards Everything

 Happiness is not perfected until it is shared
  -Jane Porter



[[alternative HTML version deleted]]

 __
 R-help@r-project.org mailing list
 https://stat.ethz.ch/mailman/listinfo/r-help
 PLEASE do read the posting guide
 http://www.R-project.org/posting-guide.htmlhttp://www.r-project.org/posting-guide.html
 and provide commented, minimal, self-contained, reproducible code.




-- 
Jim Holtman
Cincinnati, OH
+1 513 646 9390

What is the problem you are trying to solve?

[[alternative HTML version deleted]]

__
R-help@r-project.org mailing list
https://stat.ethz.ch/mailman/listinfo/r-help
PLEASE do read the posting guide http://www.R-project.org/posting-guide.html
and provide commented, minimal, self-contained, reproducible code.


Re: [R] Using library in an example

2008-06-04 Thread Duncan Murdoch

On 6/4/2008 8:09 AM, Jim Lemon wrote:

Hi all,
I've been trying to include a nice example in the Rd file for a function 
and cannot work out how to get it through R check. If I use:


if(require(maps,quietly=TRUE)) {
  ...

Warning message:
'Sys.putenv' is deprecated.
Use 'Sys.setenv' instead.
See help(Deprecated)


require() doesn't use Sys.putenv, so I'd guess some package that you're 
loading does.  However, the current version of maps doesn't, so I think 
you must be loading something that's obsolete.


sessionInfo() would have helped a lot.

Duncan Murdoch



The warning message causes R check to get the tremors and assume there 
is something wrong with the example. This also means that I can't put a 
little message telling the user that they needed maps to run the 
example, as the warning message leads to the else being detached from 
the if (and another tremor and error message).


I can't really use the following, even though it doesn't cause the 
warning (but would be messy if the user didn't have the package).


if(library(maps)) {
  ...

If I try:

if(library(maps,logical.return=TRUE)) {
  ...

I get the same warning as with require. Does this qualify as a bug in 
require/library or should I just not bother to test the result of 
library and let the user suffer?


Jim

__
R-help@r-project.org mailing list
https://stat.ethz.ch/mailman/listinfo/r-help
PLEASE do read the posting guide http://www.R-project.org/posting-guide.html
and provide commented, minimal, self-contained, reproducible code.


__
R-help@r-project.org mailing list
https://stat.ethz.ch/mailman/listinfo/r-help
PLEASE do read the posting guide http://www.R-project.org/posting-guide.html
and provide commented, minimal, self-contained, reproducible code.


Re: [R] not meaningful for factors

2008-06-04 Thread Philip Twumasi-Ankrah
It works. Thanks Jim. I guess this will be a lots of coffee morning kinda day.

jim holtman [EMAIL PROTECTED] wrote: What exactly are you trying to do?  In 
the first case you are making a logical comparison and that is legal for .  
In the second you are trying to do a logical operation () between two 
factors and that operation is not defined.  This is what the error message is 
saying.
   
 Also you first attempt is probably missing a comma:
  
  node1  - data.trt[data.trt$stage==1B   data.trt$diameter=== 4, ]



 On Wed, Jun 4, 2008 at 8:10 AM, Philip Twumasi-Ankrah [EMAIL PROTECTED] 
wrote:
 I am trying to define groupings from levels of factor variables and this the 
warning message that R give
 
  not meaningful for factors.

The nature of my task is this. I have a variable stage which has the levels 
(1B, 2A, 2B) - these are the AJCC TNM stages of cancer, and another variable 
diameter with factor levels (= 4, 4 - 6.5,  6.5; limit values are 
exclusive).
 
I am trying to define series of groupings based on these variables and others 
like them.

My first attempts were;

1. node1  - data.trt[data.trt$stage==1B   data.trt$diameter=== 4]
 2.  data.trt$stage[data.trt$stage==1B] 
data.trt$diameter[data.trt$diameter=== 4]

The second attempt was purely a fishing exercise.

R gave me the waring message:

Warning message:
 In Ops.factor(data.trt$stage[data.trt$stage == 1B], 
data.trt$diameter[data.trt$diameter ==  :
  not meaningful for factors


My question is how do I get round this and by implication is there an 
alternative way of defining a logical operation applicable to factors.
 
Thanks already for the help.

Philip


A Smile costs Nothing

But Rewards Everything

Happiness is not perfected until it is shared
 -Jane Porter
 


   [[alternative HTML version deleted]]

__
R-help@r-project.org mailing list
https://stat.ethz.ch/mailman/listinfo/r-help
 PLEASE do read the posting guide http://www.R-project.org/posting-guide.html
and provide commented, minimal, self-contained, reproducible code.
 



-- 
Jim Holtman
Cincinnati, OH
+1 513 646 9390

What is the problem you are trying to solve?  


A Smile costs Nothing  

 But Rewards Everything

Happiness is not perfected until it is shared
  -Jane Porter  
  


   
[[alternative HTML version deleted]]

__
R-help@r-project.org mailing list
https://stat.ethz.ch/mailman/listinfo/r-help
PLEASE do read the posting guide http://www.R-project.org/posting-guide.html
and provide commented, minimal, self-contained, reproducible code.


Re: [R] problems with variogram

2008-06-04 Thread Jon Olav Skoien


Laura Saltyte wrote:

Hello,

I have data at 10 locations, in each location there are time series
(T=56). Question is: when I’m fitting variogram what happens with those
measures in each location? Are they taken as repeated measures? It's very
important for my to know this

Thanks a lot
  

Hi Laura,

I think this depends on which package you use for your variogram 
function. In variogram in package gstat, you can define your wanted 
behavior by manipulating the dX-parameter. Generally, the easiest way 
you can find an answer to your question if you dont get any help from 
the help-file of your variogram function, is to look at the number of 
pairs in the calculated sample variogram;
- if you have relatively few pairs altogether (45 or less for your 10 
locations, depending on the cutoff for the variogram function), the 
variogram was calculated from the average of the observations, or from 
one of the values (probably the first)
- if the number of pairs is lower than 45*56 (the length of your time 
series), the variogram was probably calculated as an average of the 
variograms from each separate time step
- if the number of pairs is high, the variogram was probably 
calculated by comparing all observations independent of time steps with 
each other.


If you realize that your function uses the last method, I would 
recommend you to switch to one of the first two. Unless the temporal 
correlation is very high, the spatial correlation is likely to be 
(strongly) underestimated in this case.


For questions about spatial statistics, the list r-sig-geo will often be 
more useful than r-help.


Good luck,
Jon

__
R-help@r-project.org mailing list
https://stat.ethz.ch/mailman/listinfo/r-help
PLEASE do read the posting guide http://www.R-project.org/posting-guide.html
and provide commented, minimal, self-contained, reproducible code.


Re: [R] Comparing two regression lines

2008-06-04 Thread John Fox
Dear Christoph,

If I've understood properly what you intend to do, no, it doesn't make
sense. I assume from your description that you don't have two independent
samples, but rather you really have n observations and that the variables x,
y1, and y2, are measured on the same individuals. If that's correct, then
typically y1 and y2 would not be independent.

If my understanding is correct, then you should be able to do what you want
by fitting a multivariate regression and testing for the difference of the
two slopes. Of course, it must make sense to compare the slopes -- e.g., y1
and y2 must be measured in the same units. 

Here's a nonsense example illustrating how you could proceed using the
linear.hypothesis() function in the car package:

 library(car)
 mod - lm(cbind(income, education) ~ prestige, data=Duncan)
 Anova(mod)

Type II MANOVA Tests: Pillai test statistic
 Df test stat approx F num Df den DfPr(F)
prestige  1 0.828  101.216  2 42  2.2e-16 ***
---
Signif. codes:  0 '***' 0.001 '**' 0.01 '*' 0.05 '.' 0.1 ' ' 1 

 linear.hypothesis(mod, prestige, P=matrix(c(-1, 1), 2, 1))

 Response transformation matrix:
  [,1]
income  -1
education1

Sum of squares and products for the hypothesis:
 [,1]
[1,] 1048.684

Sum of squares and products for error:
 [,1]
[1,] 17828.96

Multivariate Tests: 
 Df test stat  approx F num Df den Df  Pr(F)
Pillai1 0.017 2.5292235  1 43 0.11908
Wilks 1 0.983 2.5292235  1 43 0.11908
Hotelling-Lawley  1 0.0588192 2.5292235  1 43 0.11908
Roy   1 0.0588192 2.5292235  1 43 0.11908

Note that in this case the multivariate test is really univariate.

If you apply your approach to this example, you get:

 attach(Duncan)
 y - c(income, education)
 x - c(prestige, prestige)
 f - factor(rep(c(income, education), c(45, 45)))
 mod.univ - lm(y ~ x*f)
 Anova(mod.univ)
Anova Table (Type II tests)

Response: y
  Sum Sq Df F valuePr(F)
x  46199  1 214.549  2.2e-16 ***
f   2571  1  11.938 0.0008563 ***
x:f  524  1   2.435 0.1223237
Residuals  18519 86  
---
Signif. codes:  0 '***' 0.001 '**' 0.01 '*' 0.05 '.' 0.1 ' ' 1

The sum of squares for the x:f interaction is exactly half the SS for the
hypothesis of equal slopes from the multivariate linear model, but the sum
of squares for error is off. The net result is a test statistic that is
slightly off (and has incorrect df).

I hope this helps,
 John

--
John Fox, Professor
Department of Sociology
McMaster University
Hamilton, Ontario, Canada
web: socserv.mcmaster.ca/jfox

 -Original Message-
 From: [EMAIL PROTECTED] [mailto:[EMAIL PROTECTED]
On
 Behalf Of Christoph Scherber
 Sent: June-04-08 7:37 AM
 To: [EMAIL PROTECTED]
 Subject: [R] Comparing two regression lines
 
 Dear R users,
 
 Suppose I have two different response variables y1, y2 that I regress
 separately on the same
 explanatory variable, x; sample sizes are n1=n2.
 
 Is it legitimate to compare the regression slopes (equal variances
assumed)
 by using
 
 lm(y~x*FACTOR),
 
 where FACTOR gets y1 if y1 is the response, and y2 if y2 is the
response?
 
 The problem I see here is that the residual degrees of freedom obviously
go
 up (n1-1+n2-1) although
 there were only n1=n2 true replications.
 
 On the other hand, Zar (1984) and some other statistics textbooks base
their
 calculations on a
 modified version of the t test (mainly using the residual SS from the two
 regressions), but when I
 calculated these tests by hand I found that the t-values are almost
identical
 to the one I get using
 the lm(...) approach.
 
 Part of the R script is appended below.
 
 How would you proceed?
 
 Many thanks for your help!
 
 Best wishes,
 Christoph
 
 ##
 
 lm1=lm(y1~x) # y1 and y2 are different response variables, scaled to [0;1]
 using a ranging transform
 lm2=lm(y2~x) # n is 12 for each regression
 
 model3=lm(y~x*FACTOR)
 summary(model3)
 
 sxx1=sum((y1-mean(y1))^2)
 sxx2=sum((y2-mean(y2))^2)
 
 s.pooled=(4805.2+6946.6)/20
 
 SED=sqrt(s.pooled*(1/sxx1+1/sxx2))
 
 t=(528.54-446.004)/SED #residual SS from lm1 and lm2 divided by SED
 qt(0.025,20) #compare with t distribution at 20 d.f.
 
 
 ##
 #(using R 2.6.2 on Windows XP)
 
 __
 R-help@r-project.org mailing list
 https://stat.ethz.ch/mailman/listinfo/r-help
 PLEASE do read the posting guide
http://www.R-project.org/posting-guide.html
 and provide commented, minimal, self-contained, reproducible code.

__
R-help@r-project.org mailing list
https://stat.ethz.ch/mailman/listinfo/r-help
PLEASE do read the posting guide http://www.R-project.org/posting-guide.html
and provide commented, minimal, self-contained, reproducible code.


[R] sum of unknown number of matrices

2008-06-04 Thread Shubha Vishwanath Karanth
Hi R,

 

I have a list of matrices. I need to get the sum of all the matrices in
the list.

 

Example:

a=b=c=d=matrix(1:4,2,2)

l=list(a,b,c,d)

 

 

I need:

 a+b+c+d

 [,1] [,2]

[1,]4   12

[2,]8   16

 

Something like do.call(+,l) is not working...why is this?

 

 

I may not be knowing the number of matrices in the list...

 

Thanks, Shubha

 

This e-mail may contain confidential and/or privileged i...{{dropped:13}}

__
R-help@r-project.org mailing list
https://stat.ethz.ch/mailman/listinfo/r-help
PLEASE do read the posting guide http://www.R-project.org/posting-guide.html
and provide commented, minimal, self-contained, reproducible code.


Re: [R] boxplot with text and symbols on x

2008-06-04 Thread Birgit Lemcke

Thanks Greg.

A useful solution.

Birgit

Am 02.06.2008 um 23:13 schrieb Greg Snow:

An alternative way to draw the symbols (or some approximation of  
them) is to use the my.symbols function from the TeachingDemos  
package along with ms.male and ms.female (or your improvement of  
these, also from the TeachingDemos package).  See the 4th example  
from ?ms.male (or ?ms.female).


Hope this helps,

--
Gregory (Greg) L. Snow Ph.D.
Statistical Data Center
Intermountain Healthcare
[EMAIL PROTECTED]
(801) 408-8111




-Original Message-
From: [EMAIL PROTECTED]
[mailto:[EMAIL PROTECTED] On Behalf Of Birgit Lemcke
Sent: Thursday, May 29, 2008 6:05 AM
To: Gabor Grothendieck
Cc: R Hilfe
Subject: Re: [R] boxplot with text and symbols on x

Thanks that was a huge help.

Now I am using this:

pdf(InfLengMaxVarHomogeneity.pdf)
plot(inflorescence_length_Max~Sex,xaxt=n  #unterdrückt
normale x- Achse , ylab=inflorescence length max,
main=Bartletts Homogeneity for inflorescence length
Max,data=FemMal_Sex)
Homo-bartlett.test(FemMal_Sex$inflorescence_length_Max,FemMal
_Sex$Sex)
text( 2, 500, Bartlett's K-squared=, col='red',adj = c(1,0)
) text( 2, 500, round(Homo$statistic,digits=5), col='red'
,adj = c(0,0)) text( 2, 480, df=, col='red',adj = c(1,0) )
text( 2, 480, Homo$parameter, col='red' ,adj = c(0,0))

text( 2, 460, p-value=, col='red', adj = c(1,0)) text( 2,
460, round(Homo$p.value,digits=5), col='red', adj = c(0,0))
axis(1,at=c(1,2) #Mache punkte auf Achse in Position 1 und 2
, lab=expression( \u2642, \u2640)) #zeichne dort Mann/Frau  
Symbole

dev.off()


But the symbols do not appear in the pdf. What is the reason
for that and how can I create a pdf with the symbols.

Thanks again

Birgit

Am 29.05.2008 um 13:13 schrieb Gabor Grothendieck:


1. See ?locator
2. Try this:
plot(1:2, pch = c(\u2640, \u2642))


On Thu, May 29, 2008 at 4:40 AM, Birgit Lemcke
[EMAIL PROTECTED] wrote:

Hello R-user community!

I am running R 2.7.0 on a Power Book (Tiger). (I am still R and
statistics
beginner)

I did the following :

pdf(InLnegthMaxHomogeneity.pdf)
boxplot(inflorescence_length_Max~Sex, main=Bartletts

Homogeneity for

inflorescence length,data=FemMal_Sex)
Homo-bartlett.test(FemMal_Sex$inflorescence_length_Max,FemMal_Sex
$Sex)
text( 2, 500, Bartlett's K-squared=, col='red',adj =

c(1,0) ) text(

2, 500, round(Homo$statistic,digits=5), col='red' ,adj = c
(0,0))
text( 2, 480, df=, col='red',adj = c(1,0) ) text( 2, 480,
Homo$parameter, col='red' ,adj = c(0,0))

text( 2, 460, p-value=, col='red', adj = c(1,0)) text( 2, 460,
round(Homo$p.value,digits=5), col='red', adj = c(0,0))

dev.off()


As I am still not very good in R, I guess it is not the

easiest way

to implement the text and it was kind of fiddling around

to find the

right position for the text.
I would be glad if somebody would have an easier solution.

My second question is about adding symbols (usual male and female
symbol) to
the x-axis to label the boxes.

I would be very glad if somebody could help mw with this.

Many thanks in advance.

Greets

Birgit

Birgit Lemcke
Institut für Systematische Botanik
Zollikerstrasse 107
CH-8008 Zürich
Switzerland
Ph: +41 (0)44 634 8351
[EMAIL PROTECTED]

175 Jahre UZH
«staunen.erleben.begreifen. Naturwissenschaft zum

Anfassen.» Weitere

Informationen http://www.175jahre.uzh.ch/naturwissenschaft

__
R-help@r-project.org mailing list
https://stat.ethz.ch/mailman/listinfo/r-help
PLEASE do read the posting guide http://www.R-project.org/posting-
guide.html and provide commented, minimal, self-contained,
reproducible code.



Birgit Lemcke
Institut für Systematische Botanik
Zollikerstrasse 107
CH-8008 Zürich
Switzerland
Ph: +41 (0)44 634 8351
[EMAIL PROTECTED]

175 Jahre UZH
«staunen.erleben.begreifen. Naturwissenschaft zum Anfassen.»
Weitere Informationen http://www.175jahre.uzh.ch/naturwissenschaft

__
R-help@r-project.org mailing list
https://stat.ethz.ch/mailman/listinfo/r-help
PLEASE do read the posting guide
http://www.R-project.org/posting-guide.html
and provide commented, minimal, self-contained, reproducible code.





Birgit Lemcke
Institut für Systematische Botanik
Zollikerstrasse 107
CH-8008 Zürich
Switzerland
Ph: +41 (0)44 634 8351
[EMAIL PROTECTED]

175 Jahre UZH
«staunen.erleben.begreifen. Naturwissenschaft zum Anfassen.»
Weitere Informationen http://www.175jahre.uzh.ch/naturwissenschaft

__
R-help@r-project.org mailing list
https://stat.ethz.ch/mailman/listinfo/r-help
PLEASE do read the posting guide http://www.R-project.org/posting-guide.html
and provide commented, minimal, self-contained, reproducible code.


Re: [R] sum of unknown number of matrices

2008-06-04 Thread Dimitris Rizopoulos

try a simple for loop, it will be fast enough in this case, e.g.,

matSums - function (lis) {
   out - array(data = 0, dim = dim(lis[[1]]))
   for (i in seq(along = lis))
   out - out + lis[[i]]
   out
}

a - b - c - d - matrix(1:4, 2, 2)
l - list(a, b ,c, d)

matSums(l)


I hope it helps.

Best,
Dimitris


Dimitris Rizopoulos
Biostatistical Centre
School of Public Health
Catholic University of Leuven

Address: Kapucijnenvoer 35, Leuven, Belgium
Tel: +32/(0)16/336899
Fax: +32/(0)16/337015
Web: http://med.kuleuven.be/biostat/
http://www.student.kuleuven.be/~m0390867/dimitris.htm


- Original Message - 
From: Shubha Vishwanath Karanth [EMAIL PROTECTED]

To: [EMAIL PROTECTED]
Sent: Wednesday, June 04, 2008 4:53 PM
Subject: [R] sum of unknown number of matrices



Hi R,



I have a list of matrices. I need to get the sum of all the matrices 
in

the list.



Example:

a=b=c=d=matrix(1:4,2,2)

l=list(a,b,c,d)





I need:


a+b+c+d


[,1] [,2]

[1,]4   12

[2,]8   16



Something like do.call(+,l) is not working...why is this?





I may not be knowing the number of matrices in the list...



Thanks, Shubha



This e-mail may contain confidential and/or privileged 
i...{{dropped:13}}


__
R-help@r-project.org mailing list
https://stat.ethz.ch/mailman/listinfo/r-help
PLEASE do read the posting guide 
http://www.R-project.org/posting-guide.html

and provide commented, minimal, self-contained, reproducible code.




Disclaimer: http://www.kuleuven.be/cwis/email_disclaimer.htm

__
R-help@r-project.org mailing list
https://stat.ethz.ch/mailman/listinfo/r-help
PLEASE do read the posting guide http://www.R-project.org/posting-guide.html
and provide commented, minimal, self-contained, reproducible code.


Re: [R] sum of unknown number of matrices

2008-06-04 Thread Barry Rowlingson

Shubha Vishwanath Karanth wrote:


I need:


a+b+c+d


 [,1] [,2]

[1,]4   12

[2,]8   16

 


Something like do.call(+,l) is not working...why is this?


Because do.call constructs a function call with the elements of l as 
arguments, so you end up with:


+(1:4, 1:4, 1:4, 1:4)

 but + only takes two arguments.

Use 'Reduce':

  Reduce(+,l)
  [,1] [,2]
 [1,]4   12
 [2,]8   16

 Barry

__
R-help@r-project.org mailing list
https://stat.ethz.ch/mailman/listinfo/r-help
PLEASE do read the posting guide http://www.R-project.org/posting-guide.html
and provide commented, minimal, self-contained, reproducible code.


Re: [R] sum of unknown number of matrices

2008-06-04 Thread John Fox
Dear Shubha,

This problem was coincidentally used as an illustration in the Help Desk
column in the current R News.

Actually, the brute-force method of using a loop to accumulate the sum works
quite well; a more elegant alternative, recently brought to my attention by
Kurt Hornik, uses the Reduce() function.

Here's an example:

 matrices - vector(mode=list, length=1)
 for (i in 1:1)
+   matrices[[i]] - matrix(rnorm(1), 100, 100)
 
 system.time({
+   S - matrix(0, 100, 100)
+   for (i in 1:1) S - S + matrices[[i]]
+   })
   user  system elapsed 
   0.590.000.59 
   
 system.time(S1 - Reduce(+, matrices))
   user  system elapsed 
   0.600.000.59 
 
 range(S1 - S)
[1] 0 0

I hope this helps,
 John

--
John Fox, Professor
Department of Sociology
McMaster University
Hamilton, Ontario, Canada
web: socserv.mcmaster.ca/jfox

 -Original Message-
 From: [EMAIL PROTECTED] [mailto:[EMAIL PROTECTED]
On
 Behalf Of Shubha Vishwanath Karanth
 Sent: June-04-08 10:54 AM
 To: [EMAIL PROTECTED]
 Subject: [R] sum of unknown number of matrices
 
 Hi R,
 
 
 
 I have a list of matrices. I need to get the sum of all the matrices in
 the list.
 
 
 
 Example:
 
 a=b=c=d=matrix(1:4,2,2)
 
 l=list(a,b,c,d)
 
 
 
 
 
 I need:
 
  a+b+c+d
 
  [,1] [,2]
 
 [1,]4   12
 
 [2,]8   16
 
 
 
 Something like do.call(+,l) is not working...why is this?
 
 
 
 
 
 I may not be knowing the number of matrices in the list...
 
 
 
 Thanks, Shubha
 
 
 
 This e-mail may contain confidential and/or privileged i...{{dropped:13}}
 
 __
 R-help@r-project.org mailing list
 https://stat.ethz.ch/mailman/listinfo/r-help
 PLEASE do read the posting guide
http://www.R-project.org/posting-guide.html
 and provide commented, minimal, self-contained, reproducible code.

__
R-help@r-project.org mailing list
https://stat.ethz.ch/mailman/listinfo/r-help
PLEASE do read the posting guide http://www.R-project.org/posting-guide.html
and provide commented, minimal, self-contained, reproducible code.


Re: [R] sum of unknown number of matrices

2008-06-04 Thread Shubha Vishwanath Karanth
Thanks all...Reduce() is the new function I learnt today... Thanks...

BR, Shubha
Shubha Karanth | Amba Research
Ph +91 80 3980 8031 | Mob +91 94 4886 4510 
Bangalore * Colombo * London * New York * San José * Singapore * 
www.ambaresearch.com
-Original Message-
From: Barry Rowlingson [mailto:[EMAIL PROTECTED] 
Sent: Wednesday, June 04, 2008 8:49 PM
To: Shubha Vishwanath Karanth
Cc: [EMAIL PROTECTED]
Subject: Re: [R] sum of unknown number of matrices

Shubha Vishwanath Karanth wrote:

 I need:
 
 a+b+c+d
 
  [,1] [,2]
 
 [1,]4   12
 
 [2,]8   16
 
  
 
 Something like do.call(+,l) is not working...why is this?

Because do.call constructs a function call with the elements of l as 
arguments, so you end up with:

+(1:4, 1:4, 1:4, 1:4)

  but + only takes two arguments.

Use 'Reduce':

   Reduce(+,l)
   [,1] [,2]
  [1,]4   12
  [2,]8   16

  Barry


This e-mail may contain confidential and/or privileged i...{{dropped:10}}

__
R-help@r-project.org mailing list
https://stat.ethz.ch/mailman/listinfo/r-help
PLEASE do read the posting guide http://www.R-project.org/posting-guide.html
and provide commented, minimal, self-contained, reproducible code.


Re: [R] sum of unknown number of matrices

2008-06-04 Thread Berwin A Turlach
G'day Shubha,

On Wed, 4 Jun 2008 20:23:35 +0530
Shubha Vishwanath Karanth [EMAIL PROTECTED] wrote:

 Something like do.call(+,l) is not working...why is this?

Well, as the error message says, + is either a unary or a binary
operator, i.e. it takes either one or two arguments, but not more.

 I may not be knowing the number of matrices in the list...

This is perhaps a bit complicated but it works:

R a=b=c=d=matrix(1:4,2,2)
R l=list(a,b,c,d)
R library(abind)  ## may have to install this package first
R apply(do.call(abind, list(l, along=3)), 1:2, sum)
 [,1] [,2]
[1,]4   12
[2,]8   16

HTH.

Cheers,

Berwin

=== Full address =
Berwin A TurlachTel.: +65 6515 4416 (secr)
Dept of Statistics and Applied Probability+65 6515 6650 (self)
Faculty of Science  FAX : +65 6872 3919   
National University of Singapore
6 Science Drive 2, Blk S16, Level 7  e-mail: [EMAIL PROTECTED]
Singapore 117546http://www.stat.nus.edu.sg/~statba

__
R-help@r-project.org mailing list
https://stat.ethz.ch/mailman/listinfo/r-help
PLEASE do read the posting guide http://www.R-project.org/posting-guide.html
and provide commented, minimal, self-contained, reproducible code.


[R] Constructing groupedData objects in nlme - a little problem

2008-06-04 Thread Toby Marthews
Dear R-help,

I am trying to create groupedData objects using the nlme library. I'm
missing something basic, I know:

Here is the first example in ch.1 of Pinheiro  Bates (2000):

library(nlme)
x2=Rail$travel;x1=Rail$Rail;eg1=data.frame(x1,x2);eg1gd=Rail
print(eg1gd)
x11();print(plot(eg1gd))
femodel=lm(x2~x1-1,data=eg1gd)
print(femodel$coefficients)
  Result:
   x12  x15  x11  x16  x13  x14
  31.7 50.0 54.0 82.7 84.7 96.0

...which works fine. This uses a built-in groupedData object called Rail
that is part of the nlme library.
  I am trying to 'recreate' this groupedData object. Here's what I've done:

x1=c(1,1,1,2,2,2,3,3,3,4,4,4,5,5,5,6,6,6);x2=c(55,53,54,26,37,32,78,91,85,92,100,96,49,51,50,80,85,83)
eg1=data.frame(x1,x2);colnames(eg1)=c(Rail,travel);eg1gd=groupedData(travel~1|Rail,data=eg1)
print(eg1gd)
x11();print(plot(eg1gd))
femodel=lm(x2~x1-1,data=eg1gd)
print(femodel$coefficients)
  Result:
x1
  16.49817

...but, as you can see, the coefficients I get at the end this time are
completely different and I don't know why. Somehow, I am not creating the
structure properly even though the formula and data values are all
correct.

Can anyone help? I've looked at the ?groupedData man page, but it has no
solution to this.

Thanks very much for any advice,
Toby

Pinheiro JC  Bates DM (2000). Mixed-Effects Models in S and S-PLUS (1st
ed.). Springer, New York.

__
R-help@r-project.org mailing list
https://stat.ethz.ch/mailman/listinfo/r-help
PLEASE do read the posting guide http://www.R-project.org/posting-guide.html
and provide commented, minimal, self-contained, reproducible code.


Re: [R] wildcard symbol or character

2008-06-04 Thread john.polo

Bill Venables responded and got me what i wanted with this:

[EMAIL PROTECTED] wrote:



 cv - scan(
 

 textConnection(CW-W730  CW-W720  CW-W710  CW-W700  CW-W690  CW-W680  
 	   CW-W670  CW-W660 CE-W997  CE-W987  CE-W977  CE-W967  
 			 CE-W956  CE-W944  CE-W934  CE-W924 7W-W760
 7W-W750  
 			 7W-967W-941   7W-932   7W-923   7W-914
 7W-905  
 			 7E-W565  7E-W555  7E-W545  7E-W535  7E-W525
 7E-906   
 			 7E-850   7E-840), what = )

 Read 32 items
   
  

 cv
 


  [1] CW-W730 CW-W720 CW-W710 CW-W700 CW-W690
  [6] CW-W680 CW-W670 CW-W660 CE-W997 CE-W987
 [11] CE-W977 CE-W967 CE-W956 CE-W944 CE-W934
 [16] CE-W924 7W-W760 7W-W750 7W-96   7W-941 
 [21] 7W-932  7W-923  7W-914  7W-905  7E-W565
 [26] 7E-W555 7E-W545 7E-W535 7E-W525 7E-906 
 [31] 7E-850  7E-840 
   
  

 split(cv, substring(cv, 0, 2))
 


 $`7E`
 [1] 7E-W565 7E-W555 7E-W545 7E-W535 7E-W525 7E-906 
 [7] 7E-850  7E-840 


 $`7W`
 [1] 7W-W760 7W-W750 7W-96   7W-941  7W-932  7W-923 
 [7] 7W-914  7W-905 


 $CE
 [1] CE-W997 CE-W987 CE-W977 CE-W967 CE-W956 CE-W944
 [7] CE-W934 CE-W924

 $CW
 [1] CW-W730 CW-W720 CW-W710 CW-W700 CW-W690 CW-W680
 [7] CW-W670 CW-W660


john.polo wrote:

hello all,

i want to split a list into smaller lists. the list looks like this:
CW-W730  CW-W720  CW-W710  CW-W700  CW-W690  CW-W680  CW-W670  CW-W660 
CE-W997  CE-W987  CE-W977  CE-W967  CE-W956  CE-W944  CE-W934  CE-W924 
7W-W760  7W-W750  7W-967W-941   7W-932   7W-923   7W-914   7W-905  
7E-W565  7E-W555  7E-W545  7E-W535  7E-W525  7E-906   7E-850   7E-840 ...



i want the smaller lists to be based on the first two characters, like 
CW or 7E. i tried split() where the f variable = 
c(1E-*,1W-*,2E-*,2W-*,5E-*,5W-*,7E-*,7W-*,CE-*,CW-*), 
but * doesn't work as a wildcard as i had hoped. can someone tell me 
the appropriate wildcard character/symbol to use, please?


john


__
R-help@r-project.org mailing list
https://stat.ethz.ch/mailman/listinfo/r-help
PLEASE do read the posting guide http://www.R-project.org/posting-guide.html
and provide commented, minimal, self-contained, reproducible code.


[R] mathematical expression of probability function?

2008-06-04 Thread J S

Dear R forum,
 
I have a concern regarding a mathematical expression of the probability 
function (see below). I know how to write it for only one index i, but we have 
two : i (country) and j (year). We have a set of N observations of country year 
ij (or ith country in jth year).
 
Basically, I could replace ij as a one index t and call it as a country year 
observation. Anyway, how to write this expression correctly for two indices i 
and j? Or shall we put two product signs instead?
 
The expression is similar to this:
Large Mathematical Sign of Product as a function of (Xij), 
where the low subscript of the Product is ij=1 and an upper subscript is ij=N.
 
Thanks,
Julia
 
 
_
Search that pays you back! Introducing Live Search cashback.

rchpaysyouback
[[alternative HTML version deleted]]

__
R-help@r-project.org mailing list
https://stat.ethz.ch/mailman/listinfo/r-help
PLEASE do read the posting guide http://www.R-project.org/posting-guide.html
and provide commented, minimal, self-contained, reproducible code.


Re: [R] Constructing groupedData objects in nlme - a little problem

2008-06-04 Thread Hank Stevens

HI Toby,
I think that the problem is that you have not specified using Rail and  
travel in the model statement, but rather are still using x1 and x2.  
It also does not realize that you want x1 to be a factor. Try


x1=factor(c(1,1,1,2,2,2,3,3,3,4,4,4,5,5,5,6,6,6))
x2=c(55,53,54,26,37,32,78,91,85,92,100,96,49,51,50,80,85,83)
eg1=data.frame(x1,x2);
colnames(eg1)=c(Rail,travel);
eg1gd=groupedData(travel~1|Rail,data=eg1)
print(eg1gd)
x11();
print(plot(eg1gd))

femodel=lm(x2~x1-1,data=eg1gd)
femodel=lm(travel~Rail-1,data=eg1gd)
summary(femodel)

Cheers,
Hank
On Jun 4, 2008, at 10:01 AM, Toby Marthews wrote:


Dear R-help,

I am trying to create groupedData objects using the nlme library. I'm
missing something basic, I know:

Here is the first example in ch.1 of Pinheiro  Bates (2000):

library(nlme)
x2=Rail$travel;x1=Rail$Rail;eg1=data.frame(x1,x2);eg1gd=Rail
print(eg1gd)
x11();print(plot(eg1gd))
femodel=lm(x2~x1-1,data=eg1gd)
print(femodel$coefficients)
 Result:
  x12  x15  x11  x16  x13  x14
 31.7 50.0 54.0 82.7 84.7 96.0

...which works fine. This uses a built-in groupedData object called  
Rail

that is part of the nlme library.
 I am trying to 'recreate' this groupedData object. Here's what I've  
done:


x1=c(1,1,1,2,2,2,3,3,3,4,4,4,5,5,5,6,6,6);
x2=c(55,53,54,26,37,32,78,91,85,92,100,96,49,51,50,80,85,83)
eg1=data.frame(x1,x2);
colnames(eg1)=c(Rail,travel);
eg1gd=groupedData(travel~1|Rail,data=eg1)
print(eg1gd)
x11();
print(plot(eg1gd))
femodel=lm(x2~x1-1,data=eg1gd)
print(femodel$coefficients)
 Result:
   x1
 16.49817

...but, as you can see, the coefficients I get at the end this time  
are
completely different and I don't know why. Somehow, I am not  
creating the

structure properly even though the formula and data values are all
correct.

Can anyone help? I've looked at the ?groupedData man page, but it  
has no

solution to this.

Thanks very much for any advice,
Toby

Pinheiro JC  Bates DM (2000). Mixed-Effects Models in S and S-PLUS  
(1st

ed.). Springer, New York.

__
R-help@r-project.org mailing list
https://stat.ethz.ch/mailman/listinfo/r-help
PLEASE do read the posting guide http://www.R-project.org/posting-guide.html
and provide commented, minimal, self-contained, reproducible code.




Dr. Hank Stevens, Associate Professor
338 Pearson Hall
Botany Department
Miami University
Oxford, OH 45056

Office: (513) 529-4206
Lab: (513) 529-4262
FAX: (513) 529-4243
http://www.cas.muohio.edu/~stevenmh/
http://www.cas.muohio.edu/ecology
http://www.muohio.edu/botany/

If the stars should appear one night in a thousand years, how would men
believe and adore. -Ralph Waldo Emerson, writer and philosopher  
(1803-1882)


__
R-help@r-project.org mailing list
https://stat.ethz.ch/mailman/listinfo/r-help
PLEASE do read the posting guide http://www.R-project.org/posting-guide.html
and provide commented, minimal, self-contained, reproducible code.


Re: [R] how to automatically create objects with names from a string list?

2008-06-04 Thread Greg Snow
Mark,

Others have given answers to the question that you asked, in the spirit of 
fortune(108) I am going to answer some of the questions that you should have 
asked:

Q1: Should I ever do this?
Short answer: No
Less short answer: probably not
Longer answer: You should only do this once you fully understand why you should 
never do this.

Q2: Why should I not do this?
A: There are many reasons, the most important is probably that there are better 
ways to accomplish what you want to do. Another important reason is doing this 
tends to lead to bugs that are very difficult to find and debug, these types of 
constucts lead to accidentally overwriting variables, using a different 
variable than you intend, using loops where other tools are better, etc.

Q3: What is a better method to accomplish this?
A: This depends on what all you want to do with these new variables, but for 
most cases using a list (or sometimes creating a new environment) will 
accomplish what you want much better.  For example:

 tmp - c('foo','bar','baz')
 mylist - as.list(1:3)
 names(mylist) - tmp
 mylist
$foo
[1] 1

$bar
[1] 2

$baz
[1] 3


 # or

 mylist - list()
 for(i in seq(along=tmp)){
+ mylist[[ tmp[i] ]] - i
+ }
 mylist
$foo
[1] 1

$bar
[1] 2

$baz
[1] 3


 # access a specific value
 mylist$bar
[1] 2

 # do something to all of them
 sapply( mylist, function(x) x+5 )
foo bar baz
  6   7   8

 # use more directly
 with( mylist, plot(1:10, 1:10, col=bar, pch=baz) )

 # compute new variables based on them
 mylist - within(mylist, foobar - foo + bar + baz)
 mylist
$foo
[1] 1

$bar
[1] 2

$baz
[1] 3

$foobar
[1] 6


 # save everything
 save(mylist, file='myfile')

 # remove everything in one shot
 rm(mylist)


Hope this helps,

--
Gregory (Greg) L. Snow Ph.D.
Statistical Data Center
Intermountain Healthcare
[EMAIL PROTECTED]
(801) 408-8111



 -Original Message-
 From: [EMAIL PROTECTED]
 [mailto:[EMAIL PROTECTED] On Behalf Of Mark Farnell
 Sent: Tuesday, June 03, 2008 11:16 PM
 To: R-help@r-project.org
 Subject: [R] how to automatically create objects with names
 from a string list?

 Suppose I have a string of objects names:

 name - c(foo, bar, baz)

 and I would like to use a for loop to automatically create
 three objects called foo, bar and baz accordingly.
 Then how can this be done (so that in the workspace, foo =
 1, bar = 2 and baz=3)

 for (i in name) {
 .
 }

 Thanks!

 Mark

 __
 R-help@r-project.org mailing list
 https://stat.ethz.ch/mailman/listinfo/r-help
 PLEASE do read the posting guide
 http://www.R-project.org/posting-guide.html
 and provide commented, minimal, self-contained, reproducible code.


__
R-help@r-project.org mailing list
https://stat.ethz.ch/mailman/listinfo/r-help
PLEASE do read the posting guide http://www.R-project.org/posting-guide.html
and provide commented, minimal, self-contained, reproducible code.


Re: [R] Constructing groupedData objects in nlme - a little problem

2008-06-04 Thread Dieter Menne
Toby Marthews Toby.Marthews at lsce.ipsl.fr writes:

 
 Dear R-help,
 
 I am trying to create groupedData objects using the nlme library. I'm
 missing something basic, I know:
 
 Here is the first example in ch.1 of Pinheiro  Bates (2000):
...

 
 ...but, as you can see, the coefficients I get at the end this time are
 completely different and I don't know why. Somehow, I am not creating the
 structure properly even though the formula and data values are all
 correct.
 

Hank is right. On the other hand your well-defined query points to a problem
using groupedData. I do  a lot of my work with nlme and love it, but I admit
that I only started to understand it after I totally stopped using groupedData.
Which makes reading Pinheiro/Bates sometimes difficult, because many examples
give the impression that these must be done with groupedData. Everything can be
done by explicitly saying what should be analyzed in lme and friends, there is
no need to rely on groupedData.

Note that in lme4 that partially re-designs nlme, there is no such thing as
groupedData.

Dieter

__
R-help@r-project.org mailing list
https://stat.ethz.ch/mailman/listinfo/r-help
PLEASE do read the posting guide http://www.R-project.org/posting-guide.html
and provide commented, minimal, self-contained, reproducible code.


Re: [R] Constructing groupedData objects in nlme - a little problem

2008-06-04 Thread Hank Stevens

Hi Dieter,
I never really used groupedData objects at all. I can imagine that if  
you become even more immersed in nlme and lattice, groupedData could  
reduce your typing, and sometimes simplify your life, but it was not a  
hurdle I chose willingly.

Hank
On Jun 4, 2008, at 1:15 PM, Dieter Menne wrote:


Toby Marthews Toby.Marthews at lsce.ipsl.fr writes:



Dear R-help,

I am trying to create groupedData objects using the nlme library. I'm
missing something basic, I know:

Here is the first example in ch.1 of Pinheiro  Bates (2000):

...



...but, as you can see, the coefficients I get at the end this time  
are
completely different and I don't know why. Somehow, I am not  
creating the

structure properly even though the formula and data values are all
correct.



Hank is right. On the other hand your well-defined query points to a  
problem
using groupedData. I do  a lot of my work with nlme and love it, but  
I admit
that I only started to understand it after I totally stopped using  
groupedData.
Which makes reading Pinheiro/Bates sometimes difficult, because many  
examples
give the impression that these must be done with groupedData.  
Everything can be
done by explicitly saying what should be analyzed in lme and  
friends, there is

no need to rely on groupedData.

Note that in lme4 that partially re-designs nlme, there is no such  
thing as

groupedData.

Dieter

__
R-help@r-project.org mailing list
https://stat.ethz.ch/mailman/listinfo/r-help
PLEASE do read the posting guide http://www.R-project.org/posting-guide.html
and provide commented, minimal, self-contained, reproducible code.




Dr. Hank Stevens, Associate Professor
338 Pearson Hall
Botany Department
Miami University
Oxford, OH 45056

Office: (513) 529-4206
Lab: (513) 529-4262
FAX: (513) 529-4243
http://www.cas.muohio.edu/~stevenmh/
http://www.cas.muohio.edu/ecology
http://www.muohio.edu/botany/

If the stars should appear one night in a thousand years, how would men
believe and adore. -Ralph Waldo Emerson, writer and philosopher  
(1803-1882)


__
R-help@r-project.org mailing list
https://stat.ethz.ch/mailman/listinfo/r-help
PLEASE do read the posting guide http://www.R-project.org/posting-guide.html
and provide commented, minimal, self-contained, reproducible code.


[R] estimate phase shift between two signals

2008-06-04 Thread Dylan Beaudette
Hi,

Are there any functions in R that could be used to estimate the phase-shift 
between two semi-sinusoidal vectors? Here is what I have tried so far, using 
the spectrum() function -- possibly incorrectly:


# generate some fake data, normalized to unit circle
x - jitter(seq(-2*pi, 2*pi, by=0.1), amount=pi/8)

# functions defining two out-of-phase phenomena
f1 - function(x) jitter(sin(x), amount=0.25)
f2 - function(x, a) jitter(sin(x + a), amount=0.25)

# compute y-values
# we are setting the phase shift arbitrarily
s - pi/1.5632198
y1 - f1(x)
y2 - f2(x, s)


# plot:
plot(x, y1, type='p', col='red', cex=0.5)
lines(lowess(x, y1, f=0.25), col='red')

points(x, y2, col='blue', cex=0.5)
lines(lowess(x, y2, f=0.25), col='blue')


# generate time series object
comb.ts - ts(matrix(c(y1, y2), ncol=2))

# multivariate spectral decomposition
spec - spectrum(comb.ts, detrend=FALSE)

# but how to interpret the phase estimate?
mean(spec$phase)

the mean 'phase' as returned from spectrum() does not seem to match the value 
used to generate the data... Am I mis-interpreting the use or output from 
spectrum() here? If so, is there a general procedure for estimating a 
phase-shift between two noisy signals? Would I first have to fit a smooth 
function in order to solve this analytically?

Thanks in advance,



-- 
Dylan Beaudette
Soil Resource Laboratory
http://casoilresource.lawr.ucdavis.edu/
University of California at Davis
530.754.7341

__
R-help@r-project.org mailing list
https://stat.ethz.ch/mailman/listinfo/r-help
PLEASE do read the posting guide http://www.R-project.org/posting-guide.html
and provide commented, minimal, self-contained, reproducible code.


Re: [R] estimate phase shift between two signals

2008-06-04 Thread Dylan Beaudette
On Wednesday 04 June 2008, Dylan Beaudette wrote:
 Hi,

 Are there any functions in R that could be used to estimate the phase-shift
 between two semi-sinusoidal vectors? Here is what I have tried so far,
 using the spectrum() function -- possibly incorrectly:


 # generate some fake data, normalized to unit circle
 x - jitter(seq(-2*pi, 2*pi, by=0.1), amount=pi/8)

 # functions defining two out-of-phase phenomena
 f1 - function(x) jitter(sin(x), amount=0.25)
 f2 - function(x, a) jitter(sin(x + a), amount=0.25)

 # compute y-values
 # we are setting the phase shift arbitrarily
 s - pi/1.5632198
 y1 - f1(x)
 y2 - f2(x, s)


 # plot:
 plot(x, y1, type='p', col='red', cex=0.5)
 lines(lowess(x, y1, f=0.25), col='red')

 points(x, y2, col='blue', cex=0.5)
 lines(lowess(x, y2, f=0.25), col='blue')


 # generate time series object
 comb.ts - ts(matrix(c(y1, y2), ncol=2))

 # multivariate spectral decomposition
 spec - spectrum(comb.ts, detrend=FALSE)

 # but how to interpret the phase estimate?
 mean(spec$phase)

 the mean 'phase' as returned from spectrum() does not seem to match the
 value used to generate the data... Am I mis-interpreting the use or output
 from spectrum() here? If so, is there a general procedure for estimating a
 phase-shift between two noisy signals? Would I first have to fit a smooth
 function in order to solve this analytically?

 Thanks in advance,

I should have thought about this a little bit more. Here is a brute-force 
method that may suffice for now, using nls() fit to sin(x + a).


# generate some fake data, normalized to unit circle
x - jitter(seq(-2*pi, 2*pi, by=0.1), amount=pi/8)

# functions defining two out-of-phase phenomena
f2 - function(x, a) jitter(sin(x + a), amount=0.25)

# compute y-values
# we are setting the phase shift arbitrarily
ps1 - 1.5632198
ps2 - 0.25

y1 - f2(x, ps1)
y2 - f2(x, ps2)


# plot:
plot(x, y1, type='p', col='red', cex=0.5)
lines(lowess(x, y1, f=0.25), col='red')

points(x, y2, col='blue', cex=0.5)
lines(lowess(x, y2, f=0.25), col='blue')


# generate time series object
comb.ts - ts(matrix(c(y1, y2), ncol=2))

# multivariate spectral decomposition
spec - spectrum(comb.ts, detrend=FALSE)

# but how to interpret the phase estimate?
mean(spec$phase)



# fit a simple sine function to each signal
fit1 - nls(y1 ~ sin(x + a), start=list(a=0))
fit2 - nls(y2 ~ sin(x + a), start=list(a=0))




# can we determine phase-shift by looking at zero-crossings?
# where function == 0 / changes sign

x.clean - seq(-2*pi, 2*pi, by=0.01)


y1.clean - predict(fit1, data.frame(x=x.clean))
y2.clean - predict(fit2, data.frame(x=x.clean))


plot(x.clean, y1.clean, type='l', col='red')
lines(x.clean, y2.clean, type='l', col='blue')
points(x, y1, col='red', cex=0.5)
points(x, y2, col='blue', cex=0.5)
abline(h=0, lty=2)

#compute zero-crossings
y1.zero.idx - which(abs(diff(sign(y1.clean))) == 2)
y2.zero.idx - which(abs(diff(sign(y2.clean))) == 2)


points(x.clean[y1.zero.idx], y1.clean[y1.zero.idx], pch=16, col='red')
points(x.clean[y2.zero.idx], y2.clean[y2.zero.idx], pch=16, col='blue')


# how close are they?

# estimated
mean(x.clean[y2.zero.idx] - x.clean[y1.zero.idx])
[1] 1.3625


# original phase shift
(ps1 - ps2)
[1] 1.313220


the results appear to be good enough. Any thoughts on this approach, or ideas 
on a more elegant / proper implementation?

Cheers,



-- 
Dylan Beaudette
Soil Resource Laboratory
http://casoilresource.lawr.ucdavis.edu/
University of California at Davis
530.754.7341

__
R-help@r-project.org mailing list
https://stat.ethz.ch/mailman/listinfo/r-help
PLEASE do read the posting guide http://www.R-project.org/posting-guide.html
and provide commented, minimal, self-contained, reproducible code.


[R] estimate phase shift between two signals

2008-06-04 Thread Dylan Beaudette
Hi,

Are there any functions in R that could be used to estimate the phase-shift 
between two semi-sinusoidal vectors? Here is what I have tried so far, using 
the spectrum() function -- possibly incorrectly:


# generate some fake data, normalized to unit circle
x - jitter(seq(-2*pi, 2*pi, by=0.1), amount=pi/8)

# functions defining two out-of-phase phenomena
f1 - function(x) jitter(sin(x), amount=0.25)
f2 - function(x, a) jitter(sin(x + a), amount=0.25)

# compute y-values
# we are setting the phase shift arbitrarily
s - pi/1.5632198
y1 - f1(x)
y2 - f2(x, s)


# plot:
plot(x, y1, type='p', col='red', cex=0.5)
lines(lowess(x, y1, f=0.25), col='red')

points(x, y2, col='blue', cex=0.5)
lines(lowess(x, y2, f=0.25), col='blue')


# generate time series object
comb.ts - ts(matrix(c(y1, y2), ncol=2))

# multivariate spectral decomposition
spec - spectrum(comb.ts, detrend=FALSE)

# but how to interpret the phase estimate?
mean(spec$phase)

the mean 'phase' as returned from spectrum() does not seem to match the value 
used to generate the data... Am I mis-interpreting the use or output from 
spectrum() here? If so, is there a general procedure for estimating a 
phase-shift between two noisy signals? Would I first have to fit a smooth 
function in order to solve this analytically?

Thanks in advance,



-- 
Dylan Beaudette
Soil Resource Laboratory
http://casoilresource.lawr.ucdavis.edu/
University of California at Davis
530.754.7341

__
R-help@r-project.org mailing list
https://stat.ethz.ch/mailman/listinfo/r-help
PLEASE do read the posting guide http://www.R-project.org/posting-guide.html
and provide commented, minimal, self-contained, reproducible code.


[R] Creating a simple Radar/Spider Plot from Statgraphics

2008-06-04 Thread Albert Crosby
I'm new to R - and trying to create a plot similiar to the spider plot at 
http://www.statgraphics.com/eda.htm#radar .  

I can't figure out several things... most of which I would think would be 
straightforward

How can I change the lines for each series plotted instead of creating a 
filled area?

How can I get the labels for each of the radial axes at the outside of the 
plot instead of on a small circle in the middle?

Is there an easy way to get a legend like the sample plot?

Here's my attempt so far, using the USJudgeRating dataset... I'm sure that 
there's something simple that I'm missing

 stars(USJudgeRatings[1:2,1:9],locations = 0:1, scale=FALSE, 
draw.segments=FALSE,
 col.segments=1:5,col.stars=0,key.loc=0:1,
  main=Radar/Spider Plot\nSee 
http://www.statgraphics.com/eda.htm#radar,full=TRUE)

What I really want to do is plot a dataset like this one:

Year Below   Basic   Proficient   Advanced
2005 10  25  30   35
2006 8   21  41   30
2007 15  24  48   14
2008 14  19  42   25

and get a graphic with 4 superimposed stars of different colors superimposed 
showing the shape of the distribution for each year.  The data is 
percentsges of students scoring in each category, summing to 100.  There are 
actually scores for Literacy and Math, so I might display 8 radii instead of 
the 4 above. And the dataset/graph would be repeated for comparing various 
subpopulations.

Any pointers would be welcomed!

Thanks!

Albert Crosby
Springdale Public Schools
Springdale, Arkansas

[[alternative HTML version deleted]]

__
R-help@r-project.org mailing list
https://stat.ethz.ch/mailman/listinfo/r-help
PLEASE do read the posting guide http://www.R-project.org/posting-guide.html
and provide commented, minimal, self-contained, reproducible code.


[R] possible bug in flexclust

2008-06-04 Thread Gary Lewis
Hi - Writing to see if someone can suggest whether a problem warrants a bug
report. It concerns the use of stepFlexclust in the flexclust package. The
problem concerns the size of clusters returned.

Versions: R-2.7.0 on Windows XP; RODBC_1.2-3

code snippet:
r8 - stepFlexclust(df,8,nrep=100,FUN=kcca, family=kccaFamily(kmedians))
summary(r8)  ## returns cluster sizes of 51, 115, 218, 73, 118, 140, 311,
and 118
trev - data.frame([EMAIL PROTECTED])
sqlSave(ch, trev, rownames=unitid)

in MySQL:
select r8cluster, count(*) from trev group by 1 order by 1   ## returns
cluster sizes of 51, 116, 217, 73, 117, 140, 312, 118

problem:
4 of the 8 clusters differ in sizes. They're close but I'd think they should
be exact.


So ... bug or not a bug? It'd be nice if I was just doing something wrong.

Thanks.

Gary

[[alternative HTML version deleted]]

__
R-help@r-project.org mailing list
https://stat.ethz.ch/mailman/listinfo/r-help
PLEASE do read the posting guide http://www.R-project.org/posting-guide.html
and provide commented, minimal, self-contained, reproducible code.


[R] Lattice + Word: Changing .wmf files to .pdf files

2008-06-04 Thread Jim Price

My apriori apologies if this is felt to be the wrong list for this issue -
although it starts with R, it's a combination of programs that creates the
problem.

Currently we are using windows metafile format for in-text tables for
reports created in Word. However, we've discovered some artifactual lines
being created in our final output once the Word document is changed to PDF.
The process is as follows:

# Simplest example code that contains all the sample features that create
the problem
library(lattice)

test - expand.grid(type = paste('Type', 1:4), success = c('Yes','No'))
test$result - exp(rnorm(nrow(test)))

win.metafile('u:\\test.wmf')
dotplot(
success ~ result | type, 
data = test, 
panel = function(...)
{
panel.xyplot(...)
panel.abline(v = 0, lty = 2)
},
xlim = c(-10, 10)
)
dev.off()

This is then imported into Word 2008 (running on XP Pro 2002 with service
pack 2) using the import picture tool. The resulting file is then saved as a
PDF from Word.

Although the .wmf file (and R console output if sent to there) look exactly
as you'd expect, there are a series of diagonal lines running from the
bottom of the panel.abline created index line to the upper bound of each
panel.

I confess to being somewhat confused - if windows metafile formats and PDF
are both vector graphic formats, I'm not sure where this extra line has come
from. I suspect it may be an artifact from Word's PDF creating routine, but
have no concrete proof of this.

If any one can shed enlightenment (or suggest an alternative approach that
preserves decent fidelity - we've struggled with postscript files in the
past, but this may be our opportunity to try them out again), it would be
much appreciated.


 sessionInfo()
R version 2.7.0 (2008-04-22) 
i386-pc-mingw32 

locale:
LC_COLLATE=English_United States.1252;LC_CTYPE=English_United
States.1252;LC_MONETARY=English_United
States.1252;LC_NUMERIC=C;LC_TIME=English_United States.1252

attached base packages:
[1] stats graphics  grDevices utils datasets  methods   base 

other attached packages:
[1] lattice_0.17-6

loaded via a namespace (and not attached):
[1] grid_2.7.0



Jim Price.
Cardiome Pharma Corp.
-- 
View this message in context: 
http://www.nabble.com/Lattice-%2B-Word%3A-Changing-.wmf-files-to-.pdf-files-tp17651766p17651766.html
Sent from the R help mailing list archive at Nabble.com.

__
R-help@r-project.org mailing list
https://stat.ethz.ch/mailman/listinfo/r-help
PLEASE do read the posting guide http://www.R-project.org/posting-guide.html
and provide commented, minimal, self-contained, reproducible code.


[R] dotchart

2008-06-04 Thread Nair, Murlidharan T
I am trying to plot the following data using dotchart

intersect.data-structure(list(X = structure(c(1L, 3L, 8L, 9L, 10L, 11L, 12L,
13L, 14L, 15L, 2L, 4L, 5L, 6L, 7L), .Label = c(1-100, 1001-1100,
101-200, 1101-1200, 1201-1300, 1301-1400, 1401-1500,
201-300, 301-400, 401-500, 501-600, 601-700, 701-800,
801-900, 901-1000), class = factor), svmT.mcmT = c(10L,
9L, 5L, 7L, 10L, 6L, 7L, 6L, 7L, 7L, 7L, 7L, 10L, 9L, 8L), svmT.svmCell = c(4L,
5L, 8L, 6L, 3L, 4L, 7L, 10L, 6L, 5L, 6L, 9L, 10L, 8L, 6L), svmT.mcmCell = c(12L,
11L, 4L, 8L, 7L, 11L, 5L, 9L, 5L, 7L, 7L, 6L, 6L, 8L, 5L), mcmT.svmCell = c(1L,
6L, 7L, 7L, 7L, 6L, 5L, 9L, 14L, 4L, 2L, 6L, 3L, 7L, 4L), mcmT.mcmCell = c(6L,
7L, 12L, 9L, 13L, 7L, 11L, 4L, 8L, 11L, 11L, 12L, 4L, 15L, 16L
), svmCell.mcmCell = c(5L, 9L, 7L, 7L, 8L, 6L, 3L, 9L, 4L, 13L,
9L, 10L, 5L, 7L, 7L)), .Names = c(X, svmT.mcmT, svmT.svmCell,
svmT.mcmCell, mcmT.svmCell, mcmT.mcmCell, svmCell.mcmCell
), class = data.frame, row.names = c(NA, -15L))

dotchart(intersect.data)


It gives me an error saying x must be numeric.  I can edit it manually  looking 
at the VADeaths example but is there an easy way to handle this?

Thanks ../Murli

__
R-help@r-project.org mailing list
https://stat.ethz.ch/mailman/listinfo/r-help
PLEASE do read the posting guide http://www.R-project.org/posting-guide.html
and provide commented, minimal, self-contained, reproducible code.


Re: [R] merge two data sets

2008-06-04 Thread kayj

since they are not in data1 , I do not want them to be in the merge result.



Moshe Olshansky-2 wrote:
 
 Where do you want to place ID's which are in data2 but NOT in data1?
 
 
 --- On Wed, 4/6/08, kayj [EMAIL PROTECTED] wrote:
 
 From: kayj [EMAIL PROTECTED]
 Subject: [R]  merge two data sets
 To: r-help@r-project.org
 Received: Wednesday, 4 June, 2008, 9:45 AM
 I would like to merge “data1 “that contains 100 unique
 ID’s with  another
 data set “data 2” with 150 ID’s and the age of those
 individuals ( the ID in
 data1 is a subset of the ID in data 2) I  would like to
 merge these data1
 with data2 and have the result of the merge to have the ID
 ordered as in
 data1.
 
 Can this be done in R?
 
 
 -- 
 View this message in context:
 http://www.nabble.com/merge-two-data-sets-tp17636278p17636278.html
 Sent from the R help mailing list archive at Nabble.com.
 
 __
 R-help@r-project.org mailing list
 https://stat.ethz.ch/mailman/listinfo/r-help
 PLEASE do read the posting guide
 http://www.R-project.org/posting-guide.html
 and provide commented, minimal, self-contained,
 reproducible code.
 
 __
 R-help@r-project.org mailing list
 https://stat.ethz.ch/mailman/listinfo/r-help
 PLEASE do read the posting guide
 http://www.R-project.org/posting-guide.html
 and provide commented, minimal, self-contained, reproducible code.
 
 

-- 
View this message in context: 
http://www.nabble.com/merge-two-data-sets-tp17636278p17652782.html
Sent from the R help mailing list archive at Nabble.com.

__
R-help@r-project.org mailing list
https://stat.ethz.ch/mailman/listinfo/r-help
PLEASE do read the posting guide http://www.R-project.org/posting-guide.html
and provide commented, minimal, self-contained, reproducible code.


Re: [R] merging two data sets with no column header

2008-06-04 Thread kayj



Thanks a lot for your help, I appreciate it.




David Winsemius wrote:
 
 Thanks for your reply. 
 
 In your last step If  I create a duplicate ( two similar records )
 # create a duplicate 
 vv[8,1] - 7
 vv[8,2]-'g'
 
 and then I merge vv with vv2 ,both duplicates are merged. Is there a way
 to tell R to merge only the unique records.
 
 --
 
 I don't think merge will do what you are asking. Suspect you would need to
 do it either before the merge or after. I tried using the Extract operator
 on a merged dataset, but only succeeded in omitting the duplicated items, 
 not both instances.
 
 -- 
 David Winsemius
 
 
 
 

-- 
View this message in context: 
http://www.nabble.com/merging-two-data-sets-with-no-column-header-tp17613296p17652963.html
Sent from the R help mailing list archive at Nabble.com.

__
R-help@r-project.org mailing list
https://stat.ethz.ch/mailman/listinfo/r-help
PLEASE do read the posting guide http://www.R-project.org/posting-guide.html
and provide commented, minimal, self-contained, reproducible code.


Re: [R] [Rd] An issue happens Embed R and redirt png file

2008-06-04 Thread Dirk Eddelbuettel


Wrong list, so re-directing to r-help. Consider r-sig-debian for Debian
questions too, but subscribe or else your posts bounce.

On 4 June 2008 at 07:18, Rongrong wrote:
| 
| I am a new R user and I have a question of embedding R to generate png
| On Debian, I installed R by source code.
| 
| Now I want to embed R in an application which name is MyApp, and run
| the following syntax:
| png(filename=/tmp/Rplot%03d.png)
| example(rect)
| dev.off()
|  
| Unfortunately I get the following error message:
| 
| Error in X11(paste(png::, filename, sep = ), width, height, pointsize, 
| : 
|   unable to start device PNG 
| Calls: png 
| In addition: Warning message: 
| In png(filename = /tmp/test%03d.png) : 
|   unable to open connection to X11 display ''

See the R FAQ.
 
| But I could run above syntax from R, and the result is correct, I can get
| two png files from /tmp/ folder.
|   
| I try to respectively run same syntax(the syntax is:
| capabilities()[[png]])
| under two modes, the result is different:
| In R the result is TRUE, but when MyApp embed R the result is FALSE

Because in case you have an x11 display, and in the other your don't. No
display, no x11.
 
| Then I used bitmap(file=/tmp/test) when embed R in MyApp, but get another
| error message:
| 
| Error in bitmap(file = /tmp/test) : sorry, 'gs' cannot be found

Fix that via 
$ sudo apt-get install ghostscript
 
| Besides, I have installed xvfb. What I need do?

Almost there. Wrap your call to MyApp in svfb-run, ie do

$ xvfb-run MyApp [other parameters]

and you get the virtual x11 device that png needs.   

R 2.7.0 should also alleviate the need for an x11 device due to its new
cairo-based device code.  While you didn't tell as which R version you are
running, I suspect it is not the one the current one.

Dirk

-- 
Three out of two people have difficulties with fractions.

__
R-help@r-project.org mailing list
https://stat.ethz.ch/mailman/listinfo/r-help
PLEASE do read the posting guide http://www.R-project.org/posting-guide.html
and provide commented, minimal, self-contained, reproducible code.


Re: [R] dotchart

2008-06-04 Thread Nair, Murlidharan T
Thanks! I put the labels as follows

dotchart(as.matrix(intersect.data[-1]), labels=intersect.data[,1], cex=0.8)

Murli


From: Henrique Dallazuanna [EMAIL PROTECTED]
Sent: Wednesday, June 04, 2008 1:49 PM
To: Nair, Murlidharan T
Cc: r-help@r-project.org
Subject: Re: [R] dotchart

Is because your first column and the object is a data.frame:

 dotchart(as.matrix(intersect.data[-1]))

On Wed, Jun 4, 2008 at 3:42 PM, Nair, Murlidharan T [EMAIL 
PROTECTED]mailto:[EMAIL PROTECTED] wrote:
I am trying to plot the following data using dotchart

intersect.data-structure(list(X = structure(c(1L, 3L, 8L, 9L, 10L, 11L, 12L,
13L, 14L, 15L, 2L, 4L, 5L, 6L, 7L), .Label = c(1-100, 1001-1100,
101-200, 1101-1200, 1201-1300, 1301-1400, 1401-1500,
201-300, 301-400, 401-500, 501-600, 601-700, 701-800,
801-900, 901-1000), class = factor), svmT.mcmT = c(10L,
9L, 5L, 7L, 10L, 6L, 7L, 6L, 7L, 7L, 7L, 7L, 10L, 9L, 8L), svmT.svmCell = c(4L,
5L, 8L, 6L, 3L, 4L, 7L, 10L, 6L, 5L, 6L, 9L, 10L, 8L, 6L), svmT.mcmCell = c(12L,
11L, 4L, 8L, 7L, 11L, 5L, 9L, 5L, 7L, 7L, 6L, 6L, 8L, 5L), mcmT.svmCell = c(1L,
6L, 7L, 7L, 7L, 6L, 5L, 9L, 14L, 4L, 2L, 6L, 3L, 7L, 4L), mcmT.mcmCell = c(6L,
7L, 12L, 9L, 13L, 7L, 11L, 4L, 8L, 11L, 11L, 12L, 4L, 15L, 16L
), svmCell.mcmCell = c(5L, 9L, 7L, 7L, 8L, 6L, 3L, 9L, 4L, 13L,
9L, 10L, 5L, 7L, 7L)), .Names = c(X, svmT.mcmT, svmT.svmCell,
svmT.mcmCell, mcmT.svmCell, mcmT.mcmCell, svmCell.mcmCell
), class = data.frame, row.names = c(NA, -15L))

dotchart(intersect.data)


It gives me an error saying x must be numeric.  I can edit it manually  looking 
at the VADeaths example but is there an easy way to handle this?

Thanks ../Murli

__
R-help@r-project.orgmailto:R-help@r-project.org mailing list
https://stat.ethz.ch/mailman/listinfo/r-help
PLEASE do read the posting guide http://www.R-project.org/posting-guide.html
and provide commented, minimal, self-contained, reproducible code.



--
Henrique Dallazuanna
Curitiba-Paraná-Brasil
25° 25' 40 S 49° 16' 22 O

__
R-help@r-project.org mailing list
https://stat.ethz.ch/mailman/listinfo/r-help
PLEASE do read the posting guide http://www.R-project.org/posting-guide.html
and provide commented, minimal, self-contained, reproducible code.


[R] ggplot questions

2008-06-04 Thread Thompson, David (MNR)
Hello,

A few questions about the following examples:
1. Why do the two plotting versions not produce the same result?
2. Is the 'scale_x_continuous' (or *_y_* or *_*_discrete) geom the best
way to setup grids (as in visual guide-lines) in polar (or for that
matter, any) coordinate system?
3. Why do these commands appear to generate 3 plot pages each?
4. Perhaps more questions to follow ;-)

### the data
test - structure(list(oplt = c(1L, 1L, 1L, 1L, 1L, 1L, 1L, 1L, 1L, 1L
), rplt = 1:10, az = c(57L, 94L, 96L, 152L, 182L, 185L, 227L, 
264L, 332L, 354L), dist = c(4.09, 2.8, 7.08, 7.09, 3.28, 7.85, 
6.12, 1.97, 7.68, 7.9)), .Names = c(oplt, rplt, az, dist
), row.names = c(NA, 10L), class = data.frame)

labs - structure(list(oplt = c(0, 0, 0, 0, 0), rplt = structure(c(3L, 
1L, 4L, 5L, 2L), .Label = c(E, N, o, S, W), class = factor),

az = c(0, 90, 180, 270, 360), dist = c(0, 16, 16, 16, 16)), .Names =
c(oplt, 
rplt, az, dist), row.names = c(NA, -5L), class = data.frame)

### plot version 1, setup plot structure first, add 'data' points later
ggplot() + coord_polar() + 
layer( data = labs, mapping = aes(x = az, y = dist, label = rplt), geom
= text) +
scale_x_continuous(breaks=c(90, 180, 270, 360)) +
layer( data = test, mapping = aes(x = az, y = dist, label = rplt), geom
= text)

### plot version 2, try the same all in one step
ggplot() + coord_polar() + 
layer( data = test, mapping = aes(x = az, y = dist, label = rplt), geom
= text) +
scale_x_continuous(breaks=c(90, 180, 270, 360), labels=c('90', '180',
'270', '360'))

### the scenario
I am generating graphics to show the physical layout of a forestry
research experiment.
There are 54 'cut blocks' (64x64m, 0.4ha) each containing 1 circular
'overstory plot' (16m radius, 0.08ha).
Each 'overstory plot' (oplt) contains 10 circular 'regeneration plots'
(rplt) (0.56m radius, 1m^2 or 1.13m radius, 4m^2), these are the label
data points plotted below, later to have the 1m^2 or 4m^2 rplt outline
added.
Later, each individual tree will be plotted across each oplt in the
views below and each rplt in expanded views.
All locations (rplt centers, tree positions) are recorded as azimuth and
distance from their respective plot centers, hence my inclination to use
polar coordinates.

Thanx, DaveT.
*
Silviculture Data Analyst
Ontario Forest Research Institute
Ontario Ministry of Natural Resources
[EMAIL PROTECTED]
http://ontario.ca/ofri

__
R-help@r-project.org mailing list
https://stat.ethz.ch/mailman/listinfo/r-help
PLEASE do read the posting guide http://www.R-project.org/posting-guide.html
and provide commented, minimal, self-contained, reproducible code.


Re: [R] How to solve a non-linear system of equations using R

2008-06-04 Thread Ravi Varadhan
Jorge,

You can use the package BB to try and solve this problem.

I have re-written your functions a little bit.

 # --
# Constants
# --

l=1
m=0.4795
s=0.4795

# --
# Functions to estimate f_i-k_i
# --

myfn - function(d){
d1 - d[1]
d2 - d[2]
d3 - d[3]
d4 - d[4]
res - rep(NA, 4)
res[1] -
2*d1+2*sqrt(2)*d1*d2+2*sqrt(3)*d2*d3+4*d3*d4-l*m*(1+d1^2+d2^2+d3^2+d4^2)
res[2] -
2*sqrt(2)*d2+2*d1^2+2*sqrt(6)*d1*d3+4*d2^2+4*sqrt(3)*d2*d4+6*d3^2+8*d4^2-l*(
m^2+m^3*s^(-1))*(1+d1^2+d2^2+d3^2+d4^2)
res[3] -
6*d1+12*sqrt(2)*d1*d2+18*sqrt(3)*d2*d3+48*d3*d4+2*sqrt(6)*d3+4*sqrt(6)*d1*d4
-l*(m^3+3*m^4*s^(-1)+3*m^6*s^(-2))*(1+d1^2+d2^2+d3^2+d4^2)
res[4] -
12*sqrt(2)*d2+12*d1^2+36*d2^2+72*d3^2+120*d4^2+20*sqrt(6)*d1*d3+56*sqrt(3)*d
2*d4+4*sqrt(6)*d4-l*((m^4+6*m^6*s^(-1)+15*m^6*s^(-2)+15*m^7*s^(-3))-3*l^(2)*
(m^2+m^3*s^(-1))^2)*(1+d1^2+d2^2+d3^2+d4^2)
res
}

myfn.opt - function(d){
# re-writing myfn to be used for minimization
d1 - d[1]
d2 - d[2]
d3 - d[3]
d4 - d[4]
res - rep(NA, 4)
res[1] -
2*d1+2*sqrt(2)*d1*d2+2*sqrt(3)*d2*d3+4*d3*d4-l*m*(1+d1^2+d2^2+d3^2+d4^2)
res[2] -
2*sqrt(2)*d2+2*d1^2+2*sqrt(6)*d1*d3+4*d2^2+4*sqrt(3)*d2*d4+6*d3^2+8*d4^2-l*(
m^2+m^3*s^(-1))*(1+d1^2+d2^2+d3^2+d4^2)
res[3] -
6*d1+12*sqrt(2)*d1*d2+18*sqrt(3)*d2*d3+48*d3*d4+2*sqrt(6)*d3+4*sqrt(6)*d1*d4
-l*(m^3+3*m^4*s^(-1)+3*m^6*s^(-2))*(1+d1^2+d2^2+d3^2+d4^2)
res[4] -
12*sqrt(2)*d2+12*d1^2+36*d2^2+72*d3^2+120*d4^2+20*sqrt(6)*d1*d3+56*sqrt(3)*d
2*d4+4*sqrt(6)*d4-l*((m^4+6*m^6*s^(-1)+15*m^6*s^(-2)+15*m^7*s^(-3))-3*l^(2)*
(m^2+m^3*s^(-1))^2)*(1+d1^2+d2^2+d3^2+d4^2)
sum(res^2)
}

library(BB)
 
p0 - runif(4, -1,0)
ans1 - dfsane(par=p0, fn=myfn)
ans2 - spg(par=p0, fn=myfn.opt)
ans1
ans2


Note that the above does not produce a redual of zero, so the system can't
be solved exactly.  I tried a large number of random starting values without
improving upon the solution provided by spg.  So, you may want to check
your system for its correctness.

Hope this helps,
Ravi.

-Original Message-
From: [EMAIL PROTECTED] [mailto:[EMAIL PROTECTED] On
Behalf Of Jorge Ivan Velez
Sent: Tuesday, June 03, 2008 5:09 PM
To: R mailing list
Subject: [R] How to solve a non-linear system of equations using R

Dear R-list members,

I've had a hard time trying to solve a non-linear system (nls) of equations
which structure for the equation i, i=1,...,4, is as follows:


f_i(d_1,d_2,d_3,d_4)-k_i(l,m,s) = 0(1)


In the expression above, both f_i and k_i are known functions and l, m and s
are known constants. I would like to estimate the vector d=(d_1,d_2,d_3,d_4)
which is solution of (1). Functions in R to estimate f_i-k_i are at the end
of this message.

Any help/suggestions/comments would be greatly appreciated.

Thanks in advance,

Jorge


# --
# Constants
# --

l=1
m=0.4795
s=0.4795

# --
# Functions to estimate f_i-k_i
# --

f1=function(d){
d1=d[1]
d2=d[2]
d3=d[3]
d4=d[4]
res=2*d1+2*sqrt(2)*d1*d2+2*sqrt(3)*d2*d3+4*d3*d4-l*m*(1+d1^2+d2^2+d3^2+d4^2)
res
}

f2=function(d){
d1=d[1]
d2=d[2]
d3=d[3]
d4=d[4]
res=2*sqrt(2)*d2+2*d1^2+2*sqrt(6)*d1*d3+4*d2^2+4*sqrt(3)*d2*d4+6*d3^2+8*d4^2
-l*(m^2+m^3*s^(-1))*(1+d1^2+d2^2+d3^2+d4^2)
res
}

f3=function(d){
d1=d[1]
d2=d[2]
d3=d[3]
d4=d[4]
res=6*d1+12*sqrt(2)*d1*d2+18*sqrt(3)*d2*d3+48*d3*d4+2*sqrt(6)*d3+4*sqrt(6)*d
1*d4-l*(m^3+3*m^4*s^(-1)+3*m^6*s^(-2))*(1+d1^2+d2^2+d3^2+d4^2)
res
}


f4=function(d){
d1=d[1]
d2=d[2]
d3=d[3]
d4=d[4]
res=12*sqrt(2)*d2+12*d1^2+36*d2^2+72*d3^2+120*d4^2+20*sqrt(6)*d1*d3+56*sqrt(
3)*d2*d4+4*sqrt(6)*d4-l*((m^4+6*m^6*s^(-1)+15*m^6*s^(-2)+15*m^7*s^(-3))-3*l^
(2)*(m^2+m^3*s^(-1))^2)*(1+d1^2+d2^2+d3^2+d4^2)
res
}

[[alternative HTML version deleted]]

__
R-help@r-project.org mailing list
https://stat.ethz.ch/mailman/listinfo/r-help
PLEASE do read the posting guide http://www.R-project.org/posting-guide.html
and provide commented, minimal, self-contained, reproducible code.

__
R-help@r-project.org mailing list
https://stat.ethz.ch/mailman/listinfo/r-help
PLEASE do read the posting guide http://www.R-project.org/posting-guide.html
and provide commented, minimal, self-contained, reproducible code.


[R] converting a table to a dataframe or a matrix

2008-06-04 Thread markleeds


can someone show me how to convert a table to a data.frame or a matrix ? 
I tried below and as.data.frame rearranges the columns
similarly to a melt from reshape and as.matrix didn't change it. I 
actually would prefer to change it to a dataframe but if someone
can show me how to convert it to a matrix,  then as.data.frame will work 
on that. I was thinking about just changing the class to matrix
since table looks very similar to a matrix but I wasn't sure if that 
would cause side injuries.  thanks.


temp - structure(c(0.345901639344262, 0.360117302052786, 
0.354304635761589,
0.354319180087848, 0.365524402907580, 0.359455958549223, 
0.340284360189573,
0.369763205828780, 0.304918032786885, 0.304985337243402, 
0.310596026490066,
0.287457296242069, 0.285565939771547, 0.264248704663212, 
0.297630331753555,
0.231329690346084, 0.349180327868852, 0.334897360703812, 
0.335099337748344,
0.358223523670083, 0.348909657320872, 0.376295336787565, 
0.362085308056872,
0.398907103825137), class = table, .Dim = c(8L, 3L), .Dimnames = 
structure(list(

c((-1,-0.75], (-0.75,-0.5], (-0.5,-0.25], (-0.25,0],
(0,0.25], (0.25,0.5], (0.5,0.75], (0.75,1]), c(0,
1, 2)), .Names = c(, )))

print(temp)

tempa - as.data.frame(temp)
tempb - as.matrix(temp)

print(str(tempa))
print(str(tempb))

__
R-help@r-project.org mailing list
https://stat.ethz.ch/mailman/listinfo/r-help
PLEASE do read the posting guide http://www.R-project.org/posting-guide.html
and provide commented, minimal, self-contained, reproducible code.


Re: [R] converting a table to a dataframe or a matrix

2008-06-04 Thread Henrique Dallazuanna
Hi Mark,

Try this:

as.data.frame.matrix(temp)

or

as.data.frame(unclass(temp))


On Wed, Jun 4, 2008 at 4:20 PM, [EMAIL PROTECTED] wrote:


 can someone show me how to convert a table to a data.frame or a matrix ? I
 tried below and as.data.frame rearranges the columns
 similarly to a melt from reshape and as.matrix didn't change it. I actually
 would prefer to change it to a dataframe but if someone
 can show me how to convert it to a matrix,  then as.data.frame will work on
 that. I was thinking about just changing the class to matrix
 since table looks very similar to a matrix but I wasn't sure if that would
 cause side injuries.  thanks.

 temp - structure(c(0.345901639344262, 0.360117302052786,
 0.354304635761589,
 0.354319180087848, 0.365524402907580, 0.359455958549223, 0.340284360189573,
 0.369763205828780, 0.304918032786885, 0.304985337243402, 0.310596026490066,
 0.287457296242069, 0.285565939771547, 0.264248704663212, 0.297630331753555,
 0.231329690346084, 0.349180327868852, 0.334897360703812, 0.335099337748344,
 0.358223523670083, 0.348909657320872, 0.376295336787565, 0.362085308056872,
 0.398907103825137), class = table, .Dim = c(8L, 3L), .Dimnames =
 structure(list(
c((-1,-0.75], (-0.75,-0.5], (-0.5,-0.25], (-0.25,0],
(0,0.25], (0.25,0.5], (0.5,0.75], (0.75,1]), c(0,
1, 2)), .Names = c(, )))

 print(temp)

 tempa - as.data.frame(temp)
 tempb - as.matrix(temp)

 print(str(tempa))
 print(str(tempb))

 __
 R-help@r-project.org mailing list
 https://stat.ethz.ch/mailman/listinfo/r-help
 PLEASE do read the posting guide
 http://www.R-project.org/posting-guide.html
 and provide commented, minimal, self-contained, reproducible code.




-- 
Henrique Dallazuanna
Curitiba-Paraná-Brasil
25° 25' 40 S 49° 16' 22 O

[[alternative HTML version deleted]]

__
R-help@r-project.org mailing list
https://stat.ethz.ch/mailman/listinfo/r-help
PLEASE do read the posting guide http://www.R-project.org/posting-guide.html
and provide commented, minimal, self-contained, reproducible code.


Re: [R] converting a table to a dataframe or a matrix

2008-06-04 Thread Gabor Grothendieck
Either of these will do it:

head(temp, Inf)

class(temp) - matrix

On Wed, Jun 4, 2008 at 3:20 PM,  [EMAIL PROTECTED] wrote:

 can someone show me how to convert a table to a data.frame or a matrix ? I
 tried below and as.data.frame rearranges the columns
 similarly to a melt from reshape and as.matrix didn't change it. I actually
 would prefer to change it to a dataframe but if someone
 can show me how to convert it to a matrix,  then as.data.frame will work on
 that. I was thinking about just changing the class to matrix
 since table looks very similar to a matrix but I wasn't sure if that would
 cause side injuries.  thanks.

 temp - structure(c(0.345901639344262, 0.360117302052786, 0.354304635761589,
 0.354319180087848, 0.365524402907580, 0.359455958549223, 0.340284360189573,
 0.369763205828780, 0.304918032786885, 0.304985337243402, 0.310596026490066,
 0.287457296242069, 0.285565939771547, 0.264248704663212, 0.297630331753555,
 0.231329690346084, 0.349180327868852, 0.334897360703812, 0.335099337748344,
 0.358223523670083, 0.348909657320872, 0.376295336787565, 0.362085308056872,
 0.398907103825137), class = table, .Dim = c(8L, 3L), .Dimnames =
 structure(list(
c((-1,-0.75], (-0.75,-0.5], (-0.5,-0.25], (-0.25,0],
(0,0.25], (0.25,0.5], (0.5,0.75], (0.75,1]), c(0,
1, 2)), .Names = c(, )))

 print(temp)

 tempa - as.data.frame(temp)
 tempb - as.matrix(temp)

 print(str(tempa))
 print(str(tempb))

 __
 R-help@r-project.org mailing list
 https://stat.ethz.ch/mailman/listinfo/r-help
 PLEASE do read the posting guide http://www.R-project.org/posting-guide.html
 and provide commented, minimal, self-contained, reproducible code.


__
R-help@r-project.org mailing list
https://stat.ethz.ch/mailman/listinfo/r-help
PLEASE do read the posting guide http://www.R-project.org/posting-guide.html
and provide commented, minimal, self-contained, reproducible code.


[R] Reading selected lines in an .html file

2008-06-04 Thread vittorio
Dear friend, 

In an R program running permanently on a server I would like to read hour by 
hour the temperature in *C and the humidity from a  site like this (actually, 
from many of such sites):

http://www.wunderground.com/global/stations/16239.html

How can I read the content of the site and select the info I need?

Ciao
Vittorio

__
R-help@r-project.org mailing list
https://stat.ethz.ch/mailman/listinfo/r-help
PLEASE do read the posting guide http://www.R-project.org/posting-guide.html
and provide commented, minimal, self-contained, reproducible code.


Re: [R] Lattice book

2008-06-04 Thread vittorio
Il Thursday 01 May 2008 01:07:13 Charilaos Skiadas ha scritto:
 Actually it's been out for a couple of weeks now at least. I just
 finished my first reading of it, and I must say it was spectacular.
 Congratulations Deepayan, the book gave me exactly the kind of
 lattice knowledge I needed, and then some. The graphics are really
 impressive and good illustrations of what lattice can do, and I found
 the writing very clear, with the complexity increasing at just the
 right speed. I definitely recommend it to anyone who wants to learn
 how to use lattice, at any level they desire.

 Ok, I guess I better stop now. Happy reading Mark!

 Haris Skiadas
 Department of Mathematics and Computer Science
 Hanover College

I'm half way through the reading of the book and share your view: extremely 
well done and self-explaining-. The many examples are all very clear.

 Congratulations Deepayan.

Ciao from Rome
Vittorio

__
R-help@r-project.org mailing list
https://stat.ethz.ch/mailman/listinfo/r-help
PLEASE do read the posting guide http://www.R-project.org/posting-guide.html
and provide commented, minimal, self-contained, reproducible code.


Re: [R] Packfor library-problem downloading

2008-06-04 Thread tub78
Jubilo,

I believe we have the same problem.  For a hacky solution, please see
my post
http://groups.google.com/group/r-help-archive/browse_thread/thread/bbd4ef5857dc99ef

- Stu


On May 22, 11:05 am, Jubilo [EMAIL PROTECTED] wrote:
 Hello,

 I'm a new user to R, and I was trying to install packfor in my R console
 and I always run into an error.
 I tried to search, in in the forum, someone that already might have made the
 same question, but all I could find was a general information how to install
 libraries in R.

 Pass through all that procedure (PackageData-Package Installer-Local
 Source Package[because my library was in a folder in my computer]), I have a
 message that is DONE, but then I do the command library(packfor) and I
 get an error message:

 --- 
 --WARNING:ignoringenvironment value ofR_HOME
 * Installing *binary* package 'packfor' ...
 * DONE (packfor)

  library(packfor)

 packfor: R Package for Forward Selection (Canoco Manual p.49)
 version 0.0-8Error in dyn.load(file, DLLpath = DLLpath, ...) :
   unable to load shared library
 '/Library/Frameworks/R.framework/Versions/2.7/Resources/library/packfor/lib 
 s/i386/packfor.so':

 dlopen(/Library/Frameworks/R.framework/Versions/2.7/Resources/library/packf 
 or/libs/i386/packfor.so,
 6): Library not loaded: /usr/local/lib/libgfortran.2.dylib
   Referenced from:
 /Library/Frameworks/R.framework/Versions/2.7/Resources/library/packfor/libs 
 /i386/packfor.so
   Reason: image not found
 Error in library(packfor) : .First.lib failed for 'packfor'
 --- 
 ---

 I have a mac where I installed R2.7.0, and I want to use the
 packfor_0.0-8_for_R2.7.tar.gz. I already uninstalled R twice and installed
 it again, I already used another packfor file, from another person... I ran
 out of ideas so I would like a little bit of help.

 My R also presents an other error when it starts:
 -
 During startup -Warningmessages:
 1: Setting LC_CTYPE failed, using C
 2: Setting LC_COLLATE failed, using C
 3: Setting LC_TIME failed, using C
 4: Setting LC_MESSAGES failed, using CWARNING: You're using a non-UTF8 
 locale, therefore only ASCII characters
 will work.
 Please read R for Mac OS X FAQ (see Help) section 9 and adjust your system
 preferences accordingly.
 --
 I don't know if this is important to solve the issue, but anyway I already
 done other analysis (which means that I used other libraries) and I didn't
 have any problem. Besides the section 9 from the Help talks about version
 previous the one that I have (1.0.1, 2.0 and 2.1).

 Thanks in advance for any idea that you might have ,

 Ju
 --
 View this message in 
 context:http://www.nabble.com/Packfor-library-problem-downloading-tp17406291p...
 Sent from the R help mailing list archive at Nabble.com.

         [[alternative HTML version deleted]]

 __
 [EMAIL PROTECTED] mailing listhttps://stat.ethz.ch/mailman/listinfo/r-help
 PLEASE do read the posting guidehttp://www.R-project.org/posting-guide.html
 and provide commented, minimal, self-contained, reproducible code.

__
R-help@r-project.org mailing list
https://stat.ethz.ch/mailman/listinfo/r-help
PLEASE do read the posting guide http://www.R-project.org/posting-guide.html
and provide commented, minimal, self-contained, reproducible code.


[R] problem pasting into Illustrator CS2

2008-06-04 Thread Courtney Lee Meier
To Whom it May Concern:
I have been using R version 2.6.2 for awhile now, installed on a  
PowerMac G5 running OS 10.4.11  I typically get Quartz graphics  
output from R into Adobe Illustrator CS2 simply by copying and  
pasting. Upon upgrading to R version 2.7.0, I now receive an error  
from Illustrator when trying to paste R generated graphics from the  
Quartz window into Illustrator. I do not have the exact text of the  
error message, but the general message is that the object I am trying  
to paste is too large, and won't fit on the Illustrator drawing  
board.  There are no options for reducing the size.  I re-installed R  
2.6.2 and I am now able to copy and paste my R graphics into  
Illustrator as before.  Any ideas as to why this is happening or what  
I should do if I want to be able to copy and paste into Illustrator  
using the current version of R?

Cheers,
Courtney L. Meier


-
Mr. Courtney L. Meier, PhD
Dept. Ecology  Evolutionary Biology, UCB 334
University of Colorado Boulder
Boulder, CO 80309-0334
Tel: 303-492-2557
Fax: 303-492-8699
http://ucsu.colorado.edu/~meiercl/




[[alternative HTML version deleted]]

__
R-help@r-project.org mailing list
https://stat.ethz.ch/mailman/listinfo/r-help
PLEASE do read the posting guide http://www.R-project.org/posting-guide.html
and provide commented, minimal, self-contained, reproducible code.


[R] array of arrays

2008-06-04 Thread Michael Prince
Dear R users,

I want to calculate the bias and variance of an estimator for several values
of two parameters a and b.

For example :

b1  b2
a1 bias   bias
 variance variance
a2 bias   bias
 variance variance

Can one do array of arrays ? I have tried and it did not work.

Actually I would like to get this array for several values of the number of
observations. That would be an array of arrays of arrays :-)

Maybe there is a better way to do that...

Is there also a way to export arrays to Latex ?

Thank you very much

[[alternative HTML version deleted]]

__
R-help@r-project.org mailing list
https://stat.ethz.ch/mailman/listinfo/r-help
PLEASE do read the posting guide http://www.R-project.org/posting-guide.html
and provide commented, minimal, self-contained, reproducible code.


[R] Superscript/Subscript in main title

2008-06-04 Thread Tariq Perwez
I have been trying to figure out how to get superscript/subscript in the
main title for a plot. I have tried various approaches and suggestions but
none of them work. I am trying to get the following as the main title of my
plot:

Emission of CO2 with time

(but note that 2 is subscript.)

I have tried

plot(main=Emission of Cexpression(O[2]) with time)

and I get error message. I have also tried

plot(main=Emission of Cquote(O[2]) with time)

I have also tried

plot(main)
mtext(Emission of Cexpression(O[2]) with time)

I keep getting errors. I have searched the archives but none of the threads
seem to touch upon this. I would appreciate any suggestion. Regards,

Tariq

[[alternative HTML version deleted]]

__
R-help@r-project.org mailing list
https://stat.ethz.ch/mailman/listinfo/r-help
PLEASE do read the posting guide http://www.R-project.org/posting-guide.html
and provide commented, minimal, self-contained, reproducible code.


Re: [R] Superscript/Subscript in main title

2008-06-04 Thread Henrique Dallazuanna
Try this:

plot(1:10, main = expression(Emission~of~CO[2]~with~time))


On Wed, Jun 4, 2008 at 5:31 PM, Tariq Perwez [EMAIL PROTECTED] wrote:

 I have been trying to figure out how to get superscript/subscript in the
 main title for a plot. I have tried various approaches and suggestions but
 none of them work. I am trying to get the following as the main title of my
 plot:

 Emission of CO2 with time

 (but note that 2 is subscript.)

 I have tried

 plot(main=Emission of Cexpression(O[2]) with time)

 and I get error message. I have also tried

 plot(main=Emission of Cquote(O[2]) with time)

 I have also tried

 plot(main)
 mtext(Emission of Cexpression(O[2]) with time)

 I keep getting errors. I have searched the archives but none of the threads
 seem to touch upon this. I would appreciate any suggestion. Regards,

 Tariq

[[alternative HTML version deleted]]

 __
 R-help@r-project.org mailing list
 https://stat.ethz.ch/mailman/listinfo/r-help
 PLEASE do read the posting guide
 http://www.R-project.org/posting-guide.html
 and provide commented, minimal, self-contained, reproducible code.




-- 
Henrique Dallazuanna
Curitiba-Paraná-Brasil
25° 25' 40 S 49° 16' 22 O

[[alternative HTML version deleted]]

__
R-help@r-project.org mailing list
https://stat.ethz.ch/mailman/listinfo/r-help
PLEASE do read the posting guide http://www.R-project.org/posting-guide.html
and provide commented, minimal, self-contained, reproducible code.


Re: [R] Superscript/Subscript in main title

2008-06-04 Thread Christoph Meyer
Hi Tariq,

try:

plot(x,y,main=expression(Emission of CO[2]* with time))

Cheers,
Christoph


Wednesday, June 4, 2008, 10:31:08 PM, you wrote:

 I have been trying to figure out how to get superscript/subscript in the
 main title for a plot. I have tried various approaches and suggestions but
 none of them work. I am trying to get the following as the main title of my
 plot:

 Emission of CO2 with time

 (but note that 2 is subscript.)

 I have tried

 plot(main=Emission of Cexpression(O[2]) with time)

 and I get error message. I have also tried

 plot(main=Emission of Cquote(O[2]) with time)

 I have also tried

 plot(main)
 mtext(Emission of Cexpression(O[2]) with time)

 I keep getting errors. I have searched the archives but none of the threads
 seem to touch upon this. I would appreciate any suggestion. Regards,

 Tariq

 [[alternative HTML version deleted]]

 __
 R-help@r-project.org mailing list
 https://stat.ethz.ch/mailman/listinfo/r-help
 PLEASE do read the posting guide
 http://www.R-project.org/posting-guide.html
 and provide commented, minimal, self-contained, reproducible code.



***
Dr. Christoph Meyer
Institute of Experimental Ecology
University of Ulm
Albert-Einstein-Allee 11
D-89069 Ulm
Germany
Phone:  ++49-(0)731-502-2675
Fax:++49-(0)731-502-2683
Mobile: ++49-(0)1577-156-7049
E-mail: [EMAIL PROTECTED]
http://www.uni-ulm.de/nawi/nawi-bio3.html

__
R-help@r-project.org mailing list
https://stat.ethz.ch/mailman/listinfo/r-help
PLEASE do read the posting guide http://www.R-project.org/posting-guide.html
and provide commented, minimal, self-contained, reproducible code.


Re: [R] merging two data sets with no column header

2008-06-04 Thread Henrique Dallazuanna
Perhaps something about like this:

 merge(vv[-c(which(duplicated(vv))-1, which(duplicated(vv))),], vv2, by=1)

On Wed, Jun 4, 2008 at 3:08 PM, kayj [EMAIL PROTECTED] wrote:




 Thanks a lot for your help, I appreciate it.




 David Winsemius wrote:
 
  Thanks for your reply.
 
  In your last step If  I create a duplicate ( two similar records )
  # create a duplicate
  vv[8,1] - 7
  vv[8,2]-'g'
 
  and then I merge vv with vv2 ,both duplicates are merged. Is there a way
  to tell R to merge only the unique records.
 
  --
 
  I don't think merge will do what you are asking. Suspect you would need
 to
  do it either before the merge or after. I tried using the Extract
 operator
  on a merged dataset, but only succeeded in omitting the duplicated items,
  not both instances.
 
  --
  David Winsemius
 
 
 
 

 --
 View this message in context:
 http://www.nabble.com/merging-two-data-sets-with-no-column-header-tp17613296p17652963.html
 Sent from the R help mailing list archive at Nabble.com.

 __
 R-help@r-project.org mailing list
 https://stat.ethz.ch/mailman/listinfo/r-help
 PLEASE do read the posting guide
 http://www.R-project.org/posting-guide.html
 and provide commented, minimal, self-contained, reproducible code.




-- 
Henrique Dallazuanna
Curitiba-Paraná-Brasil
25° 25' 40 S 49° 16' 22 O

[[alternative HTML version deleted]]

__
R-help@r-project.org mailing list
https://stat.ethz.ch/mailman/listinfo/r-help
PLEASE do read the posting guide http://www.R-project.org/posting-guide.html
and provide commented, minimal, self-contained, reproducible code.


[R] linear model with the repeated data type~

2008-06-04 Thread Manli Yan
here is the data:
 y-c(5,2,3,7,9,0,1,4,5)
id-c(1,1,6,6,7,8,15,15,19)
t-c(50,56,50,56,50,50,50,60,50)
table1-data.frame(y,id,t)//longitudinal data

the above is only part of data.
what  I want to do is to use the linear model for each id ,then get the
estimate value,like:

fit1-lm(y~t,data=table1,subset=(id==1))

but ,you can see the variable id is quite irregular,they are not arranaged
in order and many number missing,if I write a loop by using for,it will
give me a lot NA,
and for sure ,I dont want to type id=## for about 500 times
so,how to get all the esimates for each id,and exclude the NA,then record
those estimate in one table?

great thanks ~~

[[alternative HTML version deleted]]

__
R-help@r-project.org mailing list
https://stat.ethz.ch/mailman/listinfo/r-help
PLEASE do read the posting guide http://www.R-project.org/posting-guide.html
and provide commented, minimal, self-contained, reproducible code.


Re: [R] linear model with the repeated data type~

2008-06-04 Thread Henrique Dallazuanna
Try this:

f - function(x)any(is.na(coefficients(x)))
models - by(table1[c(y, t)], table1$id, FUN=lm)
models[!unlist(lapply(models, f))]



On Wed, Jun 4, 2008 at 6:20 PM, Manli Yan [EMAIL PROTECTED] wrote:

 here is the data:
  y-c(5,2,3,7,9,0,1,4,5)
 id-c(1,1,6,6,7,8,15,15,19)
 t-c(50,56,50,56,50,50,50,60,50)
 table1-data.frame(y,id,t)//longitudinal data

 the above is only part of data.
 what  I want to do is to use the linear model for each id ,then get the
 estimate value,like:

 fit1-lm(y~t,data=table1,subset=(id==1))

 but ,you can see the variable id is quite irregular,they are not
 arranaged
 in order and many number missing,if I write a loop by using for,it will
 give me a lot NA,
 and for sure ,I dont want to type id=## for about 500 times
 so,how to get all the esimates for each id,and exclude the NA,then record
 those estimate in one table?

 great thanks ~~

[[alternative HTML version deleted]]

 __
 R-help@r-project.org mailing list
 https://stat.ethz.ch/mailman/listinfo/r-help
 PLEASE do read the posting guide
 http://www.R-project.org/posting-guide.html
 and provide commented, minimal, self-contained, reproducible code.




-- 
Henrique Dallazuanna
Curitiba-Paraná-Brasil
25° 25' 40 S 49° 16' 22 O

[[alternative HTML version deleted]]

__
R-help@r-project.org mailing list
https://stat.ethz.ch/mailman/listinfo/r-help
PLEASE do read the posting guide http://www.R-project.org/posting-guide.html
and provide commented, minimal, self-contained, reproducible code.


Re: [R] problem pasting into Illustrator CS2

2008-06-04 Thread Dr. Christoph Scherber
Dear Courtney,

Are you exporting the graphs as postscript files? This is the usual way I
do it when moving graphs between R and Illustrator CS2. I´m afraid I do
not have a Mac, but I suppose CS2 runs similarly on both systems.

Best wishes
Christoph



 To Whom it May Concern:
 I have been using R version 2.6.2 for awhile now, installed on a
 PowerMac G5 running OS 10.4.11  I typically get Quartz graphics
 output from R into Adobe Illustrator CS2 simply by copying and
 pasting. Upon upgrading to R version 2.7.0, I now receive an error
 from Illustrator when trying to paste R generated graphics from the
 Quartz window into Illustrator. I do not have the exact text of the
 error message, but the general message is that the object I am trying
 to paste is too large, and won't fit on the Illustrator drawing
 board.  There are no options for reducing the size.  I re-installed R
 2.6.2 and I am now able to copy and paste my R graphics into
 Illustrator as before.  Any ideas as to why this is happening or what
 I should do if I want to be able to copy and paste into Illustrator
 using the current version of R?

 Cheers,
 Courtney L. Meier


 -
 Mr. Courtney L. Meier, PhD
 Dept. Ecology  Evolutionary Biology, UCB 334
 University of Colorado Boulder
 Boulder, CO 80309-0334
 Tel: 303-492-2557
 Fax: 303-492-8699
 http://ucsu.colorado.edu/~meiercl/




   [[alternative HTML version deleted]]

 __
 R-help@r-project.org mailing list
 https://stat.ethz.ch/mailman/listinfo/r-help
 PLEASE do read the posting guide
 http://www.R-project.org/posting-guide.html
 and provide commented, minimal, self-contained, reproducible code.


__
R-help@r-project.org mailing list
https://stat.ethz.ch/mailman/listinfo/r-help
PLEASE do read the posting guide http://www.R-project.org/posting-guide.html
and provide commented, minimal, self-contained, reproducible code.


[R] using as.POSIXlt on multiple columns or multiple objects

2008-06-04 Thread Farrel Buchinsky
If I want to convert one column to date and time format that R can
understand and manipulate then I would do this

initial$Started-as.POSIXlt(initial$Started)

But what happens if I have many such columns and I do not want to have
my code be stuffed with 10 similar lines (one for each variable aka
column aka vector aka object)

I thought I could be clear and use the list function and do this.
list(initial$Started ,initial$Finished,full.ques$started.ful
,full.ques$finished.ful) - lapply(list(initial$Started
,initial$Finished,full.ques$started.ful
,full.ques$finished.ful),as.POSIXlt)

But I got:
Error in list(initial$Started, initial$Finished, full.ques$started.ful,  :
  could not find function list-

I am clearly missing some basic R issue here. What am I missing?


--
Farrel Buchinsky

__
R-help@r-project.org mailing list
https://stat.ethz.ch/mailman/listinfo/r-help
PLEASE do read the posting guide http://www.R-project.org/posting-guide.html
and provide commented, minimal, self-contained, reproducible code.


Re: [R] surprising predicting capabilities

2008-06-04 Thread Ted Harding
On 03-Jun-08 16:16:13, chaogai wrote:
 Hi,
 I noticed the following fortune in R 2.7 and 2.6.2:
 
 fortune('Spreads')
 
 If anything, there should be a Law: Thou Shalt Not Even Think
 Of Producing A Graph That Looks Like Anything From A Spreadsheet.
-- Ted Harding (in a discussion about producing graphics)
   R-help (August 2008)
 
 Just wondering, what function and library gave this detailed
 prediction?
 Cheers

I suspect that .../site-library/fortunes/fortunes/fortunes.csv
was exported from Excel ...

Ted.


E-Mail: (Ted Harding) [EMAIL PROTECTED]
Fax-to-email: +44 (0)870 094 0861
Date: 04-Jun-08   Time: 23:28:08
-- XFMail --

__
R-help@r-project.org mailing list
https://stat.ethz.ch/mailman/listinfo/r-help
PLEASE do read the posting guide http://www.R-project.org/posting-guide.html
and provide commented, minimal, self-contained, reproducible code.


Re: [R] array of arrays

2008-06-04 Thread Yasir Kaheil

Is it possible to have one array for bias and one for variance? such as..
biasAr=

   b1 b2
a1 biasbias
a2 biasbias

and

varAr=
   b1 b2
a1 var var
a2 var var

then you can combine the two in a data frame?

thanks
y


Michael Prince wrote:
 
 Dear R users,
 
 I want to calculate the bias and variance of an estimator for several
 values
 of two parameters a and b.
 
 For example :
 
 b1  b2
 a1 bias   bias
  variance variance
 a2 bias   bias
  variance variance
 
 Can one do array of arrays ? I have tried and it did not work.
 
 Actually I would like to get this array for several values of the number
 of
 observations. That would be an array of arrays of arrays :-)
 
 Maybe there is a better way to do that...
 
 Is there also a way to export arrays to Latex ?
 
 Thank you very much
 
   [[alternative HTML version deleted]]
 
 __
 R-help@r-project.org mailing list
 https://stat.ethz.ch/mailman/listinfo/r-help
 PLEASE do read the posting guide
 http://www.R-project.org/posting-guide.html
 and provide commented, minimal, self-contained, reproducible code.
 
 


-
Yasir H. Kaheil
Catchment Research Facility
The University of Western Ontario 

-- 
View this message in context: 
http://www.nabble.com/array-of-arrays-tp17655733p17658264.html
Sent from the R help mailing list archive at Nabble.com.

__
R-help@r-project.org mailing list
https://stat.ethz.ch/mailman/listinfo/r-help
PLEASE do read the posting guide http://www.R-project.org/posting-guide.html
and provide commented, minimal, self-contained, reproducible code.


[R] create many variables at one time~

2008-06-04 Thread Manli Yan
  I need to create 100 variable ,whose name is id.1,id.2id.100
  then I need to let a vector say id-c(id.1,id.2id.100)
  any easy way to do this?
  thanks a lot~

[[alternative HTML version deleted]]

__
R-help@r-project.org mailing list
https://stat.ethz.ch/mailman/listinfo/r-help
PLEASE do read the posting guide http://www.R-project.org/posting-guide.html
and provide commented, minimal, self-contained, reproducible code.


Re: [R] create many variables at one time~

2008-06-04 Thread Henrique Dallazuanna
Try this:

for(i in 1:100)assign(sprintf(id.%d, i), value = sample(1))
id - ls(patt = id.[0-9])

On Wed, Jun 4, 2008 at 7:52 PM, Manli Yan [EMAIL PROTECTED] wrote:

  I need to create 100 variable ,whose name is id.1,id.2id.100
  then I need to let a vector say id-c(id.1,id.2id.100)
  any easy way to do this?
  thanks a lot~

[[alternative HTML version deleted]]

 __
 R-help@r-project.org mailing list
 https://stat.ethz.ch/mailman/listinfo/r-help
 PLEASE do read the posting guide
 http://www.R-project.org/posting-guide.html
 and provide commented, minimal, self-contained, reproducible code.




-- 
Henrique Dallazuanna
Curitiba-Paraná-Brasil
25° 25' 40 S 49° 16' 22 O

[[alternative HTML version deleted]]

__
R-help@r-project.org mailing list
https://stat.ethz.ch/mailman/listinfo/r-help
PLEASE do read the posting guide http://www.R-project.org/posting-guide.html
and provide commented, minimal, self-contained, reproducible code.


[R] do multiple anova tests

2008-06-04 Thread yabado

Hi

I think my question might be answered before but i can't find the correct
one.

Here is the question.

I have a data frame from a csv file which contains about 30 variables.

lit is like this. (the values are just number I arbitrary created here)

id group calorie weight height bmi co2 SBP
1  a   200 160 10030   50   90
2  a   100   100130   35   40  70
3  b60190 110   . .  .
4  b30.  .. ..
5  c .   .. .   ...
.   ..  etc


I want to use for loop or any other methods to write one anova function to
run
anova tests for each numeric variable above, so I can get about 30 anova
results. 

Can someone help me on this 

Thank you
-- 
View this message in context: 
http://www.nabble.com/do-multiple-anova-tests-tp17657995p17657995.html
Sent from the R help mailing list archive at Nabble.com.

__
R-help@r-project.org mailing list
https://stat.ethz.ch/mailman/listinfo/r-help
PLEASE do read the posting guide http://www.R-project.org/posting-guide.html
and provide commented, minimal, self-contained, reproducible code.


Re: [R] trouble installing Rmpi on 64-bit Ubuntu 8.04 with openmpi

2008-06-04 Thread tub78
The problem here is that the compiler cannot find the include files
for mpi.  Notice that the first checks that fail are:

 checking mpi.h usability... no
 checking mpi.h presence... no
 checking for mpi.h... no

One solution is to create a file named ~/.R/Makevars with the
following line:

PKG_CPPFLAGS = -I/opt/openmpi/include

... where /opt/openmpi/include/ contains the necessary mpi.h include
file.

Then, try to recompile.

Hope this helps,
- Stu



On May 6, 12:52 pm, Mark Kimpel [EMAIL PROTECTED] wrote:
 Subject pretty much says it all. I am running 64-bit Ubuntu 8.04, i.e. Hardy
 Heron, have openmpi installed, and get the following error message with
 attempted install of Rmpi. sessionInfo() follows.

 Mark

 checking for ANSI C header files... yes
 checking for sys/types.h... yes
 checking for sys/stat.h... yes
 checking for stdlib.h... yes
 checking for string.h... yes
 checking for memory.h... yes
 checking for strings.h... yes
 checking for inttypes.h... yes
 checking for stdint.h... yes
 checking for unistd.h... yes
 checking mpi.h usability... no
 checking mpi.h presence... no
 checking for mpi.h... no
 Try to find libmpi.so or libmpich.a
 checking for main in -lmpi... yes
 checking for openpty in -lutil... yes
 checking for main in -lpthread... yes
 configure: creating ./config.status
 config.status: creating src/Makevars
 ** libs
 gcc -std=gnu99 -I/home/mkimpel/R_HOME/R-patched/R-build/lib64/R/include
 -DPACKAGE_NAME=\\ -DPACKAGE_TARNAME=\\ -DPACKAGE_VERSION=\\
 -DPACKAGE_STRING=\\ -DPACKAGE_BUGREPORT=\\ -DSTDC_HEADERS=1
 -DHAVE_SYS_TYPES_H=1 -DHAVE_SYS_STAT_H=1 -DHAVE_STDLIB_H=1 -DHAVE_STRING_H=1
 -DHAVE_MEMORY_H=1 -DHAVE_STRINGS_H=1 -DHAVE_INTTYPES_H=1 -DHAVE_STDINT_H=1
 -DHAVE_UNISTD_H=1   -DUNKNOWN -fPIC -I/usr/local/include    -fpic  -g -O2 -c
 conversion.c -o conversion.o
 In file included from conversion.c:18:
 Rmpi.h:1:17: error: mpi.h: No such file or directory
 In file included from conversion.c:18:
 Rmpi.h:14: error: expected '=', ',', ';', 'asm' or '__attribute__' before
 'mpitype'
 make: *** [conversion.o] Error 1
 chmod: cannot access `/home/mkimpel/R_HOME/site-library-2.7.0/Rmpi/libs/*':
 No such file or directory
 ERROR: compilation failed for package 'Rmpi'
 ** Removing '/home/mkimpel/R_HOME/site-library-2.7.0/Rmpi'

 The downloaded packages are in
     /tmp/RtmppcK0FI/downloaded_packages
 Warning message:

  sessionInfo()

 R version 2.7.0 Patched (2008-05-04 r45620)
 x86_64-unknown-linux-gnu

 locale:
 LC_CTYPE=en_US.UTF-8;LC_NUMERIC=C;LC_TIME=en_US.UTF-8;LC_COLLATE=en_US.UTF- 
 8;LC_MONETARY=C;LC_MESSAGES=en_US.UTF-8;LC_PAPER=en_US.UTF-8;LC_NAME=C;LC_A 
 DDRESS=C;LC_TELEPHONE=C;LC_MEASUREMENT=en_US.UTF-8;LC_IDENTIFICATION=C

 attached base packages:
 [1] stats     graphics  grDevices utils     datasets  methods   base

 other attached packages:
 [1] graph_1.18.0

 loaded via a namespace (and not attached):
 [1] cluster_1.11.10 tcltk_2.7.0     tools_2.7.0

 --
 Mark W. Kimpel MD ** Neuroinformatics ** Dept. of Psychiatry
 Indiana University School of Medicine

 15032 Hunter Court, Westfield, IN 46074

 (317) 490-5129 Work,  Mobile  VoiceMail
 (317) 663-0513 Home (no voice mail please)

 **

         [[alternative HTML version deleted]]

 __
 [EMAIL PROTECTED] mailing listhttps://stat.ethz.ch/mailman/listinfo/r-help
 PLEASE do read the posting guidehttp://www.R-project.org/posting-guide.html
 and provide commented, minimal, self-contained, reproducible code.

__
R-help@r-project.org mailing list
https://stat.ethz.ch/mailman/listinfo/r-help
PLEASE do read the posting guide http://www.R-project.org/posting-guide.html
and provide commented, minimal, self-contained, reproducible code.


Re: [R] ignoring environment value of R_HOME error when installing packages

2008-06-04 Thread tub78
This bug has been resolved, after our administrator updated the path
of R_HOME
in our scripts.  I believe that the cause of trouble was the parallel
nature
of the installation, as mentioned in the following thread:

http://groups.google.com/group/r-help-archive/browse_thread/thread/632175125a7c4
16f/6a5220b73d3ac682?lnk=gstq=R_HOME#

regards
- S.

On Jun 4, 12:35 am, tub78 [EMAIL PROTECTED] wrote:
 I am troubled by what appears to be a glitch in the current
 distribution, or in
 its installation on our system.  I've traced it, and found a work-
 around.  Is
 this normal?  Is there a cleaner solution?

 The problem:

 During a package installation, the warning message WARNING: ignoring
 environment value of R_HOME from line 31 of R_HOME/bin/R is
 accidentally
 spliced into the CLINK_CPPFLAGS variable at line 606 of R_HOME/bin/
 INSTALL.
 This prevents the compilation C files.  Please note that I have not
 set the R_HOME
 environment variable; it is undefined in my shell.

 - Here are lines 29-32 from R_HOME/bin/R:

 if test -n ${R_HOME}  \
    test ${R_HOME} != ${R_HOME_DIR}; then
   echo WARNING: ignoring environment value of R_HOME
 fi

 - Here is line 606 of R_HOME/bin/INSTALL:

     CLINK_CPPFLAGS=`echo
 tools:::.find_cinclude_paths(file='DESCRIPTION') | \
       ${R_EXE} --vanilla --slave`

 The work-around:

 First some background.  The command sequence R CMD INSTALL sets in
 motion a sequence of scripts that collectively manage the installation
 process.

     R        R_HOME/bin/R              calls Rcmd script on line
 148
     CMD      R_HOME/lib/R/bin/Rcmd     calls INSTALL script on
 line 45
     INSTALL  R_HOME/lib/R/bin/INSTALL  encounters the error on
 line 606

 Now, if you are the owner of you installation, then you can probably
 just edit
 the INSTALL file directly.

 Otherwise, if you specify the full path of an executable in place of
 the word
 INSTALL on the command line, the Rcmd script will detect this and
 call your
 script instead of the normal INSTALL script (see Rcmd line 37).

     e.g.  R CMD path.to.alternate.install.script ...

 One can just copy the INSTALL script, changing line 606 to
 CLINK_CPPFLAGS = 
 and hope for the best.  If the package you are compiling specifies its
 own C
 include files, then you will have to modify the variable accordingly,
 or else
 use the ~/.R/Makevars mechanism.

 But, the question remains, is there an nicer solution?

 Thanks,
 - Stu

 __
 [EMAIL PROTECTED] mailing listhttps://stat.ethz.ch/mailman/listinfo/r-help
 PLEASE do read the posting guidehttp://www.R-project.org/posting-guide.html
 and provide commented, minimal, self-contained, reproducible code.

__
R-help@r-project.org mailing list
https://stat.ethz.ch/mailman/listinfo/r-help
PLEASE do read the posting guide http://www.R-project.org/posting-guide.html
and provide commented, minimal, self-contained, reproducible code.


[R] ARS function

2008-06-04 Thread rhead

I am having problem calling ars function in ARS package. My parameter domain
does not have upper bound or lower bound, so I did not givevalues to
parameters ubx,lbx. But it keeps telling me the starting values I provided
didn't have value to the left or to the right of the mode of the target
density. I plotted the density function values for the absicssas vs the
abscissae, the curve has a mode in the middle. Has anybody have this problem
before?Thanks!
rhead
-- 
View this message in context: 
http://www.nabble.com/ARS-function-tp17658712p17658712.html
Sent from the R help mailing list archive at Nabble.com.

__
R-help@r-project.org mailing list
https://stat.ethz.ch/mailman/listinfo/r-help
PLEASE do read the posting guide http://www.R-project.org/posting-guide.html
and provide commented, minimal, self-contained, reproducible code.


[R] merging 3 data sets at once

2008-06-04 Thread kayj

Hi All,

I am looking into merging 3 data sets I know how to do that by merging data1
with data2 and then merging the result with data 3. I was wondering if it
can be done all at once so I tried,

M-merge(data1,data2,data3, by=”ID”)

It does not work!
Any ideas?



-- 
View this message in context: 
http://www.nabble.com/merging-3-data-sets-at-once-tp17658873p17658873.html
Sent from the R help mailing list archive at Nabble.com.

__
R-help@r-project.org mailing list
https://stat.ethz.ch/mailman/listinfo/r-help
PLEASE do read the posting guide http://www.R-project.org/posting-guide.html
and provide commented, minimal, self-contained, reproducible code.


[R] Survey: Commercial R companies

2008-06-04 Thread Gregory Warnes

Hello Everyone,

In preparation for an upcoming talk, I would like to assemble a list  
of companies that provide consulting, services, products, or training  
for R.

I am already aware of a number of such companies including (in  
alphabetical order):

BlueReference http://inference.us
Insightful http://www.insightful.com
Metrum Institute http://www.metruminstitute.org
Metrum Research Group, LLC http://www.metrumrg.com
Random Technologies LLC http://random-technologies-llc.com
REvolution Computing Inc http://revolution-computing.com
Statistics.com http://www.statistics.com
XLSolutions Corporation http://www.xlsolutions-corp.com

If you are aware of a company that provides consulting, services,  
products, or training for R, please complete the form below and  
forward it directly to me at [EMAIL PROTECTED] by Wednesday June 11.   
I will collate all of the responses and post a complete list back to  
R-help.

Thanks,

-Greg

Gregory R. Warnes, Ph.D.
Associate Professor
Center for Biodefence Immune Modeling
and
Department of Biostatistics and Computational Biology
University of Rochester

=== Cut Here, Send to [EMAIL PROTECTED] ==

Company Name:

Address (including country):




Web Site:

Email Address:

Phone Number:

Type of Products and Services:

[ ]  Commercial versions of R
Product Name:

[ ]  General Training (e.g. Introduction to R, Programming in R, etc.)
Courses Taught:

[ ]  Domain-specific Training (e.g. R for PK/PD Modeling, Bayesian  
Modeling using R, etc.)
 Domain:

[ ]  Add-on packages (Off the shelf, not custom)
Package Names:

[ ]  Software/package development (custom)

[ ]  Parallel Computing Tools

[ ]  Statistical Consulting utilizing R
Area of expertise:

[ ]  GUI environments
Product Name:

[ ]  Qualification/Validation Services

[ ]  Other:



[[alternative HTML version deleted]]

__
R-help@r-project.org mailing list
https://stat.ethz.ch/mailman/listinfo/r-help
PLEASE do read the posting guide http://www.R-project.org/posting-guide.html
and provide commented, minimal, self-contained, reproducible code.


[R] quite complicated case(the repeated data arranage~)

2008-06-04 Thread Manli Yan
 Hi everyone:
 I have been struggling with this repeated data type for whole afternoon,I
sent two emails to server for help,many people kindly responded , hereby
thank you so much,but since I dont want to write to much in email,so I
divide the problem in parts,so far this seem did not work out very well,so
this is my whole problem~

 first I have example of data here:
treatment-c(low,high,high,high,high,low,low,low,low)
age-c(50,60,50,50,60,50,60,50,60)
y-c(20,40,30,11,23,24,56,65,60)
id-c(1,1,3,4,4,6,8,9,9)
table1-cbind(treatment,id,age,y)

*the actual data are way more than this*,the id is from 1~500,and not in
regular ,some number missing~
all I want to do is
put the cases to variable according the id
for example when id =1
we have
treatment1  age1   y
low   5020
high  6040
this will generate a new matrix
for this example I will have 6 new matrix,according to id.
it is reasonable to do this in loop for,but the I met some problem:
1:how to automatically generate the new title such as treatment1 and age1
until treatment 500,age500
2:as you see,id is not strictly from 1 to 500,some time it jump from 15
to19,skip 16,17,18, if I write a loop,it will give me lot NA,certainly I
need a way to avoid this

and one more,say I have 100 vectors like
x1x100,x1-c(1,3),x2-x(2,2),...x100-(number,number)
I want to combine all this 100 vectors in one new vector say Xall
which is Xall-cbind(x1,x2,...x100) //this need to type in 100
variables,take alot time
  so Xall will be
x1x2  x3 .x100
  1   2number
  3   2number
is here any easy way to do this,instead of inputting them one by one?

*and Great Thanks for your time~~*

[[alternative HTML version deleted]]

__
R-help@r-project.org mailing list
https://stat.ethz.ch/mailman/listinfo/r-help
PLEASE do read the posting guide http://www.R-project.org/posting-guide.html
and provide commented, minimal, self-contained, reproducible code.


[R] nls() newbie convergence problem

2008-06-04 Thread Bernard Leemon
I'm sure this must be a nls() newbie question, but I'm stumped.
I'm trying to do the example from Draper
and Yang (1997).  They give this snippet of S-Plus code:

Specify the weight function:
weight  - function(y,x1,x2,b0,b1,b2)
{
pred -  b0+b1*x1 + b2*x2
parms - abs(b1*b2)^(1/3)
(y-pred)/parms
}
Fit the model
gmfit  -nls(~weight(y,x1,x2,b0,b1,b2), observe,list(starting value))

in converting this to R, I left the weight function alone and replaced the
nls() with

gmfit -
 
nls(~weight(y,x1,x2,b0,b1,b2),data=dydata,trace=TRUE,start=list(b0=1,b1=1,b2=1))

where dydata is the appropriate data.frame.

nls() quickly (6 iterations) finds the exact values from Draper  Yang for
b0, b1, and b2 but
despite reporting a discrepancy of only 3.760596e-29 by the 7th iteration,
it merrily goes on
to 50 iterations and thinks it never converged.  how do I tell nls() that
I'm actually quite
happy with 3.760596e-29 and it need not work further?

thanks for any suggestions.

gary mcclelland (aka bernie)
univ of colorado

[[alternative HTML version deleted]]

__
R-help@r-project.org mailing list
https://stat.ethz.ch/mailman/listinfo/r-help
PLEASE do read the posting guide http://www.R-project.org/posting-guide.html
and provide commented, minimal, self-contained, reproducible code.


[R] using barchart in lattice package and conditioning

2008-06-04 Thread markleeds
I have the data structure below and I'm attempting to send it into 
barchart using the R code below it. I don't get an error but I don't get 
any output either. Deepyan's new Lattice book is amazing and there are 
some examples sort of similar to what i'm doing  but I couldn't see a 
way of using the formula interface to condition on what I wanted to 
condition on so I decided to use split instead.  I'm not sure if that's 
where my problem lies but if anyone has experience in using the barchart 
function in the lattice package and could take a look at what I'm trying 
to do,

it would really be appreciated. thanks.

p.s: my data set is already tallied by proportion so i don't need to use 
prop.table and i also have other additional columns in my data set.
these two issues are complicating matters for me also. i'm a lattice 
newbie.


stocks.all-structure(list(group = structure(c(1L, 1L, 1L, 1L, 1L, 1L, 
1L,

1L, 1L, 1L, 1L, 1L, 1L, 1L, 1L, 1L, 1L, 1L, 1L, 1L, 1L, 1L, 1L,
1L, 1L, 1L, 1L, 1L, 1L, 1L, 1L, 1L), .Label = AVT.NYSE, class = 
factor),

buckets = structure(c(8L, 6L, 4L, 2L, 1L, 3L, 5L, 7L, 8L,
6L, 4L, 2L, 1L, 3L, 5L, 7L, 8L, 6L, 4L, 2L, 1L, 3L, 5L, 7L,
8L, 6L, 4L, 2L, 1L, 3L, 5L, 7L), .Label = c([0,0.25, [-0.25,0],
[0.25,0.50], [-0.5,-0.25], [0.5,0.75], [-0.75,-0.50],
[0.75,1.0], [-1.0,-0.75]), class = factor), neutral = 
c(0.345901639344262,

0.360117302052786, 0.354304635761589, 0.354319180087848,
0.365524402907580, 0.359455958549223, 0.340284360189573,
0.369763205828780, 0.265100671140940, 0.250599520383693,
0.264124293785311, 0.257874015748031, 0.2520833,
0.260869565217391, 0.263257575757576, 0.341911764705882,
0.197761194029851, 0.192307692307692, 0.137724550898204,
0.184647302904564, 0.176991150442478, 0.173796791443850,
0.217573221757322, 0.181818181818182, 0.151162790697674,
0.163865546218487, 0.167346938775510, 0.159638554216867,
0.169934640522876, 0.133047210300429, 0.122093023255814,
0.223529411764706), negative = c(0.304918032786885, 
0.304985337243402,

0.310596026490066, 0.287457296242069, 0.285565939771547,
0.264248704663212, 0.297630331753555, 0.231329690346084,
0.345637583892617, 0.34652278177458, 0.353107344632768, 
0.319881889763780,

0.3416667, 0.329923273657289, 0.348484848484849,
0.301470588235294, 0.365671641791045, 0.384615384615385,
0.38622754491018, 0.369294605809129, 0.371681415929204, 
0.382352941176471,

0.338912133891213, 0.340909090909091, 0.406976744186047,
0.399159663865546, 0.355102040816327, 0.355421686746988,
0.398692810457516, 0.450643776824034, 0.337209302325581,
0.282352941176471), positive = c(0.349180327868852, 
0.334897360703812,

0.335099337748344, 0.358223523670083, 0.348909657320872,
0.376295336787565, 0.362085308056872, 0.398907103825137,
0.389261744966443, 0.402877697841727, 0.382768361581921,
0.422244094488189, 0.40625, 0.40920716112532, 0.388257575757576,
0.356617647058824, 0.436567164179104, 0.423076923076923,
0.476047904191617, 0.446058091286307, 0.451327433628319,
0.443850267379679, 0.443514644351464, 0.477272727272727,
0.441860465116279, 0.436974789915966, 0.477551020408163,
0.484939759036145, 0.431372549019608, 0.416309012875536,
0.540697674418605, 0.494117647058824), wt = structure(c(1L,
1L, 1L, 1L, 1L, 1L, 1L, 1L, 2L, 2L, 2L, 2L, 2L, 2L, 2L, 2L,
3L, 3L, 3L, 3L, 3L, 3L, 3L, 3L, 4L, 4L, 4L, 4L, 4L, 4L, 4L,
4L), .Label = c(5.0, 10.0, 20.0, 30.0), class = factor)), 
.Names = c(group,

buckets, neutral, negative, positive, wt), row.names = c(NA,
32L), class = data.frame)



# R CODE FOR BARCHART
#

temp-split(stocks.all,list(stocks.all$group,stocks.all$wt))
print(temp)

lapply(temp, function(.df){
print(barchart(.df, stack = TRUE,  auto.key = list(columns = 3)))
})

__
R-help@r-project.org mailing list
https://stat.ethz.ch/mailman/listinfo/r-help
PLEASE do read the posting guide http://www.R-project.org/posting-guide.html
and provide commented, minimal, self-contained, reproducible code.


[R] warning message for lmer model with poisson family

2008-06-04 Thread arams

Hello,

I'm trying to run an lmer model with family poisson but receive the
following
warning message: 

model-lmer(count~tem+dat+alt+year+tem:dat+tem:alt+tem:year+dat:alt+dat:year+alt:year+(1|id),family=poisson)
Error in objective(.par, ...) : 
  Leading minor of order 2 in downdated X'X is not positive definite

I'm using the newest version of R and recently installed the following
libraries: (lme4, matrix, lattice).
I don't understand what the warning message means or know how to fix it,
please help!
My response variable is a count variable (n=551); 212 with value = 0, and
the rest range between 1-3.
-- 
View this message in context: 
http://www.nabble.com/warning-message-for-lmer-model-with-poisson-family-tp17659512p17659512.html
Sent from the R help mailing list archive at Nabble.com.

__
R-help@r-project.org mailing list
https://stat.ethz.ch/mailman/listinfo/r-help
PLEASE do read the posting guide http://www.R-project.org/posting-guide.html
and provide commented, minimal, self-contained, reproducible code.


Re: [R] merging 3 data sets at once

2008-06-04 Thread William Pepe

I could suggest merge(merge(data1,data2), data3). 
 
However, one problem I notice is that is assigns age=12 and gender=M to 
everyone with id=1, and so on.
 
How are we to know that person with id=1 in data1 is the same person with id=1 
in data2 and data3? 
Bill Date: Wed, 4 Jun 2008 16:24:04 -0700 From: [EMAIL PROTECTED] To: 
r-help@r-project.org Subject: [R] merging 3 data sets at once   Hi All,  
I am looking into merging 3 data sets I know how to do that by merging data1 
with data2 and then merging the result with data 3. I was wondering if it can 
be done all at once so I tried,  M-merge(data1,data2,data3, by=”ID”)  It 
does not work! Any ideas?--  View this message in context: 
http://www.nabble.com/merging-3-data-sets-at-once-tp17658873p17658873.html 
Sent from the R help mailing list archive at Nabble.com.  
__ R-help@r-project.org mailing 
list https://stat.ethz.ch/mailman/listinfo/r-help PLEASE do read the posting 
guide http://www.R-project.org/posting-guide.html and provide commented, 
minimal, self-contained, reproducible code.
_
Instantly invite friends from Facebook and other social networks to join yo
https://www.invite2messenger.net/im/?source=TXT_EML_WLH_InviteFriends
[[alternative HTML version deleted]]

__
R-help@r-project.org mailing list
https://stat.ethz.ch/mailman/listinfo/r-help
PLEASE do read the posting guide http://www.R-project.org/posting-guide.html
and provide commented, minimal, self-contained, reproducible code.


Re: [R] merging 3 data sets at once

2008-06-04 Thread William Pepe

By the way, I used age and gender as examples. Plus, I've seen something very 
similar posted a while ago. If the problem I listed isn't an issue, then that 
code will work.
 
Best,
 
Bill


From: [EMAIL PROTECTED]: [EMAIL PROTECTED]; [EMAIL PROTECTED]: RE: [R] merging 
3 data sets at onceDate: Wed, 4 Jun 2008 20:50:34 -0400


I could suggest merge(merge(data1,data2), data3).  However, one problem I 
notice is that is assigns age=12 and gender=M to everyone with id=1, and so on. 
How are we to know that person with id=1 in data1 is the same person with id=1 
in data2 and data3? Bill Date: Wed, 4 Jun 2008 16:24:04 -0700 From: [EMAIL 
PROTECTED] To: r-help@r-project.org Subject: [R] merging 3 data sets at once 
  Hi All,  I am looking into merging 3 data sets I know how to do that by 
merging data1 with data2 and then merging the result with data 3. I was 
wondering if it can be done all at once so I tried,  
M-merge(data1,data2,data3, by=”ID”)  It does not work! Any ideas?-- 
 View this message in context: 
http://www.nabble.com/merging-3-data-sets-at-once-tp17658873p17658873.html 
Sent from the R help mailing list archive at Nabble.com.  
__ R-help@r-project.org mailing 
list https://stat.ethz.ch/mailman/listinfo/r-help PLEASE do read the posting 
guide http://www.R-project.org/posting-guide.html and provide commented, 
minimal, self-contained, reproducible code.

Instantly invite friends from Facebook and other social networks to join you on 
Windows Live™ Messenger. Invite friends now! 
_
Now you can invite friends from Facebook and other groups to join you on Wi
https://www.invite2messenger.net/im/?source=TXT_EML_WLH_AddNow_Now
[[alternative HTML version deleted]]

__
R-help@r-project.org mailing list
https://stat.ethz.ch/mailman/listinfo/r-help
PLEASE do read the posting guide http://www.R-project.org/posting-guide.html
and provide commented, minimal, self-contained, reproducible code.


Re: [R] nls() newbie convergence problem

2008-06-04 Thread Katharine Mullen

dydata
   x1 x2   y
1   9 27 248
2   9 22 213
3   4 23 190
4  11 16 183
5  -6 25 144
6  11 14 169
7  -4 13  72
8   2  8  73
9  10 13 156
10  8 30 263
11 12 10 147
12 -7  5  -2
13  0 10  75
14 12  0  77
15  9  8 115
16 12 24 245
17 34 23 370
18 12  1  84
19 10 37 324
20 26 30 371

weight - function(y,x1,x2,b0,b1,b2)
{
  pred -  b0+b1*x1 + b2*x2
  parms - abs(b1*b2)^(1/3)
  (y-pred)/parms
}

gmfit -  nls(~weight(y,x1,x2,b0,b1,b2),data=dydata,trace=TRUE,
  start=list(b0=1,b1=1,b2=1), control=list(warnOnly=TRUE))

library(minpack.lm)

weight2 - function(p, y,x1,x2)
{
  pred -  p[1]+p[2]*x1 + p[3]*x2
  parms - abs(p[2]*p[3])^(1/3)
  (y-pred)/parms
}

gmfit2 -  nls.lm(par = c(1,1,1),
  fn = weight2, y=dydata$y,x=dydata$x1,x2=dydata$x2,
  control=list(nprint=1))


dydata is from Norman R. Draper, Yonghong (Fred) Yang, Generalization
of the geometric mean functional relationship, Computational
Statistics  Data AnalysisVolume 23, Issue 3, 9 January 1997, Pages
355-372; I don't think the attachment got through.

nls uses an orthogonality convergence criterium.  Bates talks about this
in the post at
https://stat.ethz.ch/pipermail/r-devel/2000-August/021059.html and it's
also described in the book by Bates and Watts (Nonlinear regression
analysis its applications).  Apparently here the residuals are so small
they are effectively 0, and this criteria does not work; there is a
warning about using zero-residual data in the help page for nls.

nls.lm from the package minpack.lm stops if any of a few different
criteria are met; in this case it stops at the time you think is right.

On Wed, 4 Jun 2008, Bernard Leemon wrote:

 I'm sure this must be a nls() newbie question, but I'm stumped.
 I'm trying to do the example from Draper
 and Yang (1997).  They give this snippet of S-Plus code:

 Specify the weight function:
 weight  - function(y,x1,x2,b0,b1,b2)
 {
 pred -  b0+b1*x1 + b2*x2
 parms - abs(b1*b2)^(1/3)
 (y-pred)/parms
 }
 Fit the model
 gmfit  -nls(~weight(y,x1,x2,b0,b1,b2), observe,list(starting value))

 in converting this to R, I left the weight function alone and replaced the
 nls() with

 gmfit -
  
 nls(~weight(y,x1,x2,b0,b1,b2),data=dydata,trace=TRUE,start=list(b0=1,b1=1,b2=1))

 where dydata is the appropriate data.frame.

 nls() quickly (6 iterations) finds the exact values from Draper  Yang for
 b0, b1, and b2 but
 despite reporting a discrepancy of only 3.760596e-29 by the 7th iteration,
 it merrily goes on
 to 50 iterations and thinks it never converged.  how do I tell nls() that
 I'm actually quite
 happy with 3.760596e-29 and it need not work further?

 thanks for any suggestions.

 gary mcclelland (aka bernie)
 univ of colorado

   [[alternative HTML version deleted]]

 __
 R-help@r-project.org mailing list
 https://stat.ethz.ch/mailman/listinfo/r-help
 PLEASE do read the posting guide http://www.R-project.org/posting-guide.html
 and provide commented, minimal, self-contained, reproducible code.


__
R-help@r-project.org mailing list
https://stat.ethz.ch/mailman/listinfo/r-help
PLEASE do read the posting guide http://www.R-project.org/posting-guide.html
and provide commented, minimal, self-contained, reproducible code.


[R] sorting the data~

2008-06-04 Thread Manli Yan
   id-c(1,1,1,1,3,3,3,7,7,7,7,11,11,11)

how to sort  this kind of data to
   id:(1,1,1,1,2,2,2,3,3,3,3,4,4,4.)

thanks~

[[alternative HTML version deleted]]

__
R-help@r-project.org mailing list
https://stat.ethz.ch/mailman/listinfo/r-help
PLEASE do read the posting guide http://www.R-project.org/posting-guide.html
and provide commented, minimal, self-contained, reproducible code.


Re: [R] merging 3 data sets at once

2008-06-04 Thread Ben Bolker
Not following this very carefully, but today I did something similar
with 

  Reduce(merge,list(d1,d2,d3)) ...

__
R-help@r-project.org mailing list
https://stat.ethz.ch/mailman/listinfo/r-help
PLEASE do read the posting guide http://www.R-project.org/posting-guide.html
and provide commented, minimal, self-contained, reproducible code.


Re: [R] sorting the data~

2008-06-04 Thread Erik Iverson

Are these the ranks of the data?

help.search(rank)



Manli Yan wrote:

   id-c(1,1,1,1,3,3,3,7,7,7,7,11,11,11)

how to sort  this kind of data to
   id:(1,1,1,1,2,2,2,3,3,3,3,4,4,4.)

thanks~

[[alternative HTML version deleted]]

__
R-help@r-project.org mailing list
https://stat.ethz.ch/mailman/listinfo/r-help
PLEASE do read the posting guide http://www.R-project.org/posting-guide.html
and provide commented, minimal, self-contained, reproducible code.


__
R-help@r-project.org mailing list
https://stat.ethz.ch/mailman/listinfo/r-help
PLEASE do read the posting guide http://www.R-project.org/posting-guide.html
and provide commented, minimal, self-contained, reproducible code.


Re: [R] sorting the data~

2008-06-04 Thread Manli Yan
  no,the id is  variable of a table,such as:
  treatment id  age response
low 1   50   20
low 1   60   30
high5   50   30
high5   60  40

...

I want to rearranage the table according the id (increasing),since id is not
strictly from 1~n,it is in increasing order but sometime jump through many
number like 1 1 5 5,I like them to be 1 1 2 2~



2008/6/4 Erik Iverson [EMAIL PROTECTED]:

 Are these the ranks of the data?

 help.search(rank)



 Manli Yan wrote:

id-c(1,1,1,1,3,3,3,7,7,7,7,11,11,11)

 how to sort  this kind of data to
   id:(1,1,1,1,2,2,2,3,3,3,3,4,4,4.)

 thanks~

[[alternative HTML version deleted]]

 __
 R-help@r-project.org mailing list
 https://stat.ethz.ch/mailman/listinfo/r-help
 PLEASE do read the posting guide
 http://www.R-project.org/posting-guide.htmlhttp://www.r-project.org/posting-guide.html
 and provide commented, minimal, self-contained, reproducible code.



[[alternative HTML version deleted]]

__
R-help@r-project.org mailing list
https://stat.ethz.ch/mailman/listinfo/r-help
PLEASE do read the posting guide http://www.R-project.org/posting-guide.html
and provide commented, minimal, self-contained, reproducible code.


Re: [R] create many variables at one time~

2008-06-04 Thread Jorge Ivan Velez
Try also,

id- paste('id.',1:100,sep=)

HTH,

Jorge


On Wed, Jun 4, 2008 at 6:52 PM, Manli Yan [EMAIL PROTECTED] wrote:

  I need to create 100 variable ,whose name is id.1,id.2id.100
  then I need to let a vector say id-c(id.1,id.2id.100)
  any easy way to do this?
  thanks a lot~

[[alternative HTML version deleted]]

 __
 R-help@r-project.org mailing list
 https://stat.ethz.ch/mailman/listinfo/r-help
 PLEASE do read the posting guide
 http://www.R-project.org/posting-guide.html
 and provide commented, minimal, self-contained, reproducible code.


[[alternative HTML version deleted]]

__
R-help@r-project.org mailing list
https://stat.ethz.ch/mailman/listinfo/r-help
PLEASE do read the posting guide http://www.R-project.org/posting-guide.html
and provide commented, minimal, self-contained, reproducible code.


Re: [R] wildcard symbol or character

2008-06-04 Thread john.polo

Jorge Ivan Velez wrote:


Dear John,

Assuming that your information is in the list x, does

substr(x,1,2)

work for you?

HTH,

Jorge

Jorge,
i tried

split(rtt, substr(rtt,1,3))

and that worked also. (i didn't think to nest it when you first 
suggested it.) i just have to clean up the levels thing. thanks!!


now, i look into regex some...

john

__
R-help@r-project.org mailing list
https://stat.ethz.ch/mailman/listinfo/r-help
PLEASE do read the posting guide http://www.R-project.org/posting-guide.html
and provide commented, minimal, self-contained, reproducible code.


Re: [R] create many variables at one time~

2008-06-04 Thread Greg Snow
An easy and good way to make a bunch of variables is to store them in a list.  
For example:

 mylist - replicate(100, sample(1:100, 10), simplify=FALSE)
 id - paste('id.',1:100, sep='')
 names(mylist) - id
 sapply(mylist, median)
  id.1   id.2   id.3   id.4   id.5   id.6   id.7   id.8   id.9  id.10  id.11
  23.0   66.0   58.0   42.5   66.5   42.0   52.0   41.0   64.0   55.0   73.5
 id.12  id.13  id.14  id.15  id.16  id.17  id.18  id.19  id.20  id.21  id.22
  57.0   64.0   62.0   45.0   32.5   44.5   54.5   70.5   57.0   72.5   41.0
 id.23  id.24  id.25  id.26  id.27  id.28  id.29  id.30  id.31  id.32  id.33
  43.0   44.5   48.5   71.0   68.5   33.5   44.5   62.5   58.5   40.0   47.0
 id.34  id.35  id.36  id.37  id.38  id.39  id.40  id.41  id.42  id.43  id.44
  45.5   33.5   62.0   69.0   60.5   22.0   69.0   36.0   43.0   64.5   42.5
 id.45  id.46  id.47  id.48  id.49  id.50  id.51  id.52  id.53  id.54  id.55
  46.0   49.5   47.5   44.5   66.5   65.0   61.0   43.5   39.0   50.0   49.0
 id.56  id.57  id.58  id.59  id.60  id.61  id.62  id.63  id.64  id.65  id.66
  56.0   31.0   45.5   37.0   68.0   39.5   69.5   47.0   66.0   56.5   55.5
 id.67  id.68  id.69  id.70  id.71  id.72  id.73  id.74  id.75  id.76  id.77
  42.0   30.5   49.5   32.0   59.5   33.5   38.0   51.0   74.0   39.5   42.0
 id.78  id.79  id.80  id.81  id.82  id.83  id.84  id.85  id.86  id.87  id.88
  35.0   55.5   51.0   31.5   44.0   41.5   32.5   60.5   58.5   41.5   39.5
 id.89  id.90  id.91  id.92  id.93  id.94  id.95  id.96  id.97  id.98  id.99
  46.0   51.0   60.0   51.5   43.5   65.5   42.5   38.0   43.0   46.0   53.5
id.100
  41.0


Now what do you want to do with all of those variables?


From: [EMAIL PROTECTED] [EMAIL PROTECTED] On Behalf Of Manli Yan [EMAIL 
PROTECTED]
Sent: Wednesday, June 04, 2008 4:52 PM
To: r-help@r-project.org
Subject: [R] create many variables at one time~

  I need to create 100 variable ,whose name is id.1,id.2id.100
  then I need to let a vector say id-c(id.1,id.2id.100)
  any easy way to do this?
  thanks a lot~

[[alternative HTML version deleted]]

__
R-help@r-project.org mailing list
https://stat.ethz.ch/mailman/listinfo/r-help
PLEASE do read the posting guide http://www.R-project.org/posting-guide.html
and provide commented, minimal, self-contained, reproducible code.

__
R-help@r-project.org mailing list
https://stat.ethz.ch/mailman/listinfo/r-help
PLEASE do read the posting guide http://www.R-project.org/posting-guide.html
and provide commented, minimal, self-contained, reproducible code.


Re: [R] quite complicated case(the repeated data arranage~)

2008-06-04 Thread Greg Snow
For the first question, what do you want to do with all the subsets?  There are 
tools like split, by, lapply, lmList, etc. that make working with all the 
subsets easy.  If you tell us what your final goal is, we may be able to help 
with a simple solution that does not need the intermediate tables, at least 
not explicitly.

For the 2nd question try:

tmp - paste('x',1:100, sep='')
tmp2 - lapply(tmp, function(x) get(x)
Xall - do.call('cbind', tmp2)


From: [EMAIL PROTECTED] [EMAIL PROTECTED] On Behalf Of Manli Yan [EMAIL 
PROTECTED]
Sent: Wednesday, June 04, 2008 6:19 PM
To: r-help@r-project.org
Subject: [R] quite complicated case(the repeated data arranage~)

 Hi everyone:
 I have been struggling with this repeated data type for whole afternoon,I
sent two emails to server for help,many people kindly responded , hereby
thank you so much,but since I dont want to write to much in email,so I
divide the problem in parts,so far this seem did not work out very well,so
this is my whole problem~

 first I have example of data here:
treatment-c(low,high,high,high,high,low,low,low,low)
age-c(50,60,50,50,60,50,60,50,60)
y-c(20,40,30,11,23,24,56,65,60)
id-c(1,1,3,4,4,6,8,9,9)
table1-cbind(treatment,id,age,y)

*the actual data are way more than this*,the id is from 1~500,and not in
regular ,some number missing~
all I want to do is
put the cases to variable according the id
for example when id =1
we have
treatment1  age1   y
low   5020
high  6040
this will generate a new matrix
for this example I will have 6 new matrix,according to id.
it is reasonable to do this in loop for,but the I met some problem:
1:how to automatically generate the new title such as treatment1 and age1
until treatment 500,age500
2:as you see,id is not strictly from 1 to 500,some time it jump from 15
to19,skip 16,17,18, if I write a loop,it will give me lot NA,certainly I
need a way to avoid this

and one more,say I have 100 vectors like
x1x100,x1-c(1,3),x2-x(2,2),...x100-(number,number)
I want to combine all this 100 vectors in one new vector say Xall
which is Xall-cbind(x1,x2,...x100) //this need to type in 100
variables,take alot time
  so Xall will be
x1x2  x3 .x100
  1   2number
  3   2number
is here any easy way to do this,instead of inputting them one by one?

*and Great Thanks for your time~~*

[[alternative HTML version deleted]]

__
R-help@r-project.org mailing list
https://stat.ethz.ch/mailman/listinfo/r-help
PLEASE do read the posting guide http://www.R-project.org/posting-guide.html
and provide commented, minimal, self-contained, reproducible code.

__
R-help@r-project.org mailing list
https://stat.ethz.ch/mailman/listinfo/r-help
PLEASE do read the posting guide http://www.R-project.org/posting-guide.html
and provide commented, minimal, self-contained, reproducible code.


Re: [R] using barchart in lattice package and conditioning

2008-06-04 Thread Steven McKinney
Hi Mark,

I get output, after a while,
and reams of it.  Very likely
not what you wanted.

Can you describe what you are trying to
see in barcharts for these data?  It's
not obvious to me from the code below.

Best

Steve McKinney 



-Original Message-
From: [EMAIL PROTECTED] on behalf of [EMAIL PROTECTED]
Sent: Wed 6/4/2008 5:26 PM
To: r-help@r-project.org
Subject: [R] using barchart in lattice package and conditioning
 
I have the data structure below and I'm attempting to send it into 
barchart using the R code below it. I don't get an error but I don't get 
any output either. Deepyan's new Lattice book is amazing and there are 
some examples sort of similar to what i'm doing  but I couldn't see a 
way of using the formula interface to condition on what I wanted to 
condition on so I decided to use split instead.  I'm not sure if that's 
where my problem lies but if anyone has experience in using the barchart 
function in the lattice package and could take a look at what I'm trying 
to do,
it would really be appreciated. thanks.

p.s: my data set is already tallied by proportion so i don't need to use 
prop.table and i also have other additional columns in my data set.
these two issues are complicating matters for me also. i'm a lattice 
newbie.

stocks.all-structure(list(group = structure(c(1L, 1L, 1L, 1L, 1L, 1L, 
1L,
1L, 1L, 1L, 1L, 1L, 1L, 1L, 1L, 1L, 1L, 1L, 1L, 1L, 1L, 1L, 1L,
1L, 1L, 1L, 1L, 1L, 1L, 1L, 1L, 1L), .Label = AVT.NYSE, class = 
factor),
 buckets = structure(c(8L, 6L, 4L, 2L, 1L, 3L, 5L, 7L, 8L,
 6L, 4L, 2L, 1L, 3L, 5L, 7L, 8L, 6L, 4L, 2L, 1L, 3L, 5L, 7L,
 8L, 6L, 4L, 2L, 1L, 3L, 5L, 7L), .Label = c([0,0.25, [-0.25,0],
 [0.25,0.50], [-0.5,-0.25], [0.5,0.75], [-0.75,-0.50],
 [0.75,1.0], [-1.0,-0.75]), class = factor), neutral = 
c(0.345901639344262,
 0.360117302052786, 0.354304635761589, 0.354319180087848,
 0.365524402907580, 0.359455958549223, 0.340284360189573,
 0.369763205828780, 0.265100671140940, 0.250599520383693,
 0.264124293785311, 0.257874015748031, 0.2520833,
 0.260869565217391, 0.263257575757576, 0.341911764705882,
 0.197761194029851, 0.192307692307692, 0.137724550898204,
 0.184647302904564, 0.176991150442478, 0.173796791443850,
 0.217573221757322, 0.181818181818182, 0.151162790697674,
 0.163865546218487, 0.167346938775510, 0.159638554216867,
 0.169934640522876, 0.133047210300429, 0.122093023255814,
 0.223529411764706), negative = c(0.304918032786885, 
0.304985337243402,
 0.310596026490066, 0.287457296242069, 0.285565939771547,
 0.264248704663212, 0.297630331753555, 0.231329690346084,
 0.345637583892617, 0.34652278177458, 0.353107344632768, 
0.319881889763780,
 0.3416667, 0.329923273657289, 0.348484848484849,
 0.301470588235294, 0.365671641791045, 0.384615384615385,
 0.38622754491018, 0.369294605809129, 0.371681415929204, 
0.382352941176471,
 0.338912133891213, 0.340909090909091, 0.406976744186047,
 0.399159663865546, 0.355102040816327, 0.355421686746988,
 0.398692810457516, 0.450643776824034, 0.337209302325581,
 0.282352941176471), positive = c(0.349180327868852, 
0.334897360703812,
 0.335099337748344, 0.358223523670083, 0.348909657320872,
 0.376295336787565, 0.362085308056872, 0.398907103825137,
 0.389261744966443, 0.402877697841727, 0.382768361581921,
 0.422244094488189, 0.40625, 0.40920716112532, 0.388257575757576,
 0.356617647058824, 0.436567164179104, 0.423076923076923,
 0.476047904191617, 0.446058091286307, 0.451327433628319,
 0.443850267379679, 0.443514644351464, 0.477272727272727,
 0.441860465116279, 0.436974789915966, 0.477551020408163,
 0.484939759036145, 0.431372549019608, 0.416309012875536,
 0.540697674418605, 0.494117647058824), wt = structure(c(1L,
 1L, 1L, 1L, 1L, 1L, 1L, 1L, 2L, 2L, 2L, 2L, 2L, 2L, 2L, 2L,
 3L, 3L, 3L, 3L, 3L, 3L, 3L, 3L, 4L, 4L, 4L, 4L, 4L, 4L, 4L,
 4L), .Label = c(5.0, 10.0, 20.0, 30.0), class = factor)), 
.Names = c(group,
buckets, neutral, negative, positive, wt), row.names = c(NA,
32L), class = data.frame)



# R CODE FOR BARCHART
#

temp-split(stocks.all,list(stocks.all$group,stocks.all$wt))
print(temp)

lapply(temp, function(.df){
print(barchart(.df, stack = TRUE,  auto.key = list(columns = 3)))
})

__
R-help@r-project.org mailing list
https://stat.ethz.ch/mailman/listinfo/r-help
PLEASE do read the posting guide http://www.R-project.org/posting-guide.html
and provide commented, minimal, self-contained, reproducible code.

__
R-help@r-project.org mailing list
https://stat.ethz.ch/mailman/listinfo/r-help
PLEASE do read the posting guide http://www.R-project.org/posting-guide.html
and provide commented, minimal, self-contained, reproducible code.


Re: [R] warning message for lmer model with poisson family

2008-06-04 Thread Steven McKinney

Looks like you have some independent
variables in your model that correlate
perfectly so your design matrix is
not of full rank, probably because
your independent variable data is not
balanced.

Does a simpler fit such as
  model-lmer(count~tem+(1|id),family=poisson)
give you a result?

You might need to build up your model from such
a simple one, adding one new term at a time
until you find the one that causes the issue.

You probably do not have enough balanced
data to allow fitting such a complex
model.

HTH

Steve McKinney

-Original Message-
From: [EMAIL PROTECTED] on behalf of arams
Sent: Wed 6/4/2008 5:18 PM
To: r-help@r-project.org
Subject: [R]  warning message for lmer model with poisson family
 

Hello,

I'm trying to run an lmer model with family poisson but receive the
following
warning message: 

model-lmer(count~tem+dat+alt+year+tem:dat+tem:alt+tem:year+dat:alt+dat:year+alt:year+(1|id),family=poisson)
Error in objective(.par, ...) : 
  Leading minor of order 2 in downdated X'X is not positive definite

I'm using the newest version of R and recently installed the following
libraries: (lme4, matrix, lattice).
I don't understand what the warning message means or know how to fix it,
please help!
My response variable is a count variable (n=551); 212 with value = 0, and
the rest range between 1-3.
-- 
View this message in context: 
http://www.nabble.com/warning-message-for-lmer-model-with-poisson-family-tp17659512p17659512.html
Sent from the R help mailing list archive at Nabble.com.

__
R-help@r-project.org mailing list
https://stat.ethz.ch/mailman/listinfo/r-help
PLEASE do read the posting guide http://www.R-project.org/posting-guide.html
and provide commented, minimal, self-contained, reproducible code.

__
R-help@r-project.org mailing list
https://stat.ethz.ch/mailman/listinfo/r-help
PLEASE do read the posting guide http://www.R-project.org/posting-guide.html
and provide commented, minimal, self-contained, reproducible code.


Re: [R] sorting the data~

2008-06-04 Thread ctu

id-c(1,1,1,1,3,3,3,7,7,7,7,11,11,11,2,2,2,4,4,4,4,8,8,8)
sort(id)
[1]  1  1  1  1  2  2  2  3  3  3  4  4  4  4  7  7  7  7  8  8  8 11 11 11



Quoting Manli Yan [EMAIL PROTECTED]:


  no,the id is  variable of a table,such as:
  treatment id  age response
low 1   50   20
low 1   60   30
high5   50   30
high5   60  40

...

I want to rearranage the table according the id (increasing),since id is not
strictly from 1~n,it is in increasing order but sometime jump through many
number like 1 1 5 5,I like them to be 1 1 2 2~



2008/6/4 Erik Iverson [EMAIL PROTECTED]:


Are these the ranks of the data?

help.search(rank)



Manli Yan wrote:


   id-c(1,1,1,1,3,3,3,7,7,7,7,11,11,11)

how to sort  this kind of data to
  id:(1,1,1,1,2,2,2,3,3,3,3,4,4,4.)

thanks~

   [[alternative HTML version deleted]]

__
R-help@r-project.org mailing list
https://stat.ethz.ch/mailman/listinfo/r-help
PLEASE do read the posting guide
http://www.R-project.org/posting-guide.htmlhttp://www.r-project.org/posting-guide.html
and provide commented, minimal, self-contained, reproducible code.





[[alternative HTML version deleted]]

__
R-help@r-project.org mailing list
https://stat.ethz.ch/mailman/listinfo/r-help
PLEASE do read the posting guide http://www.R-project.org/posting-guide.html
and provide commented, minimal, self-contained, reproducible code.



__
R-help@r-project.org mailing list
https://stat.ethz.ch/mailman/listinfo/r-help
PLEASE do read the posting guide http://www.R-project.org/posting-guide.html
and provide commented, minimal, self-contained, reproducible code.


Re: [R] linear model in the repeated data type~

2008-06-04 Thread Manli Yan
hi:lot thanks,how to use list to extract,I type  allFit$coefficents,it came
to nothing,
such as I need to extract the estimates,how to do it by using list

2008/6/3 Austin, Matt [EMAIL PROTECTED]:

 How about


 library(nlme)
 allFits - lmList(y ~ t|id, data=table1, pool=FALSE)

 or

 allFits - by(table1, table1$id, function(x) lm(y ~ t, data=x))

 Both ways store the results as a list, so you can access individual results
 using list extraction.


 --Matt

 -Original Message-
 From: [EMAIL PROTECTED] [mailto:[EMAIL PROTECTED]
 On Behalf Of Manli Yan
 Sent: Tuesday, June 03, 2008 9:07 PM
 To: r-help@r-project.org
 Subject: [R] linear model in the repeated data type~

  here is the data:
  y-c(5,2,3,7,9,0,1,4,5)
 id-c(1,1,6,6,7,8,15,15,19)
 t-c(50,56,50,56,50,50,50,60,50)
 table1-data.frame(y,id,t)//longitudinal data

 what  I want to do is to use the linear model for each id ,then get the
 estimate value,like:

 fit1-lm(y~t,data=table1,subset=(id==1))

 but ,you can see the variable id is quite irregular,they are not
 arranaged in order and many number missing,if I write a loop by using
 for,it will give me a lot NA, and for sure ,I dont want to type id=##
 for about 500 times,any one know how to deal with it?

[[alternative HTML version deleted]]

 __
 R-help@r-project.org mailing list
 https://stat.ethz.ch/mailman/listinfo/r-help
 PLEASE do read the posting guide
 http://www.R-project.org/posting-guide.htmlhttp://www.r-project.org/posting-guide.html
 and provide commented, minimal, self-contained, reproducible code.


[[alternative HTML version deleted]]

__
R-help@r-project.org mailing list
https://stat.ethz.ch/mailman/listinfo/r-help
PLEASE do read the posting guide http://www.R-project.org/posting-guide.html
and provide commented, minimal, self-contained, reproducible code.


  1   2   >